Указать знак производных функций: «Как определить знаки производной функции?» – Яндекс.Кью

Содержание

Исследование функции. Задание В15 (2014)

В этой статье я продолжу объяснение решения задач на  исследование  функции с помощью производной из задания В15, а именно задач на нахождение максимума и минимума функции.

Я хочу показать решение задач, которые вызывают наибольшие затруднения у моих учеников.

Чтобы найти максимум или минимум функции, нужно следовать такому алгоритму:

1. Найти производную функции.

2. Приравнять ее к нулю.

3. С помощью метода интервалов  определить промежутки, на которых производная сохраняет знак.

4. Если нам нужно найти точку максимума функции, ищем точку, в которой производная меняет знак с “+” на “-“.

Если нам нужно найти точку минимума функции, ищем точку, в которой производная меняет знак с “-” на “+”.

 

1. Задание B15 (№ 26726)

Найдите точку максимума функции  .

1. Найдем производную функции. Для начала удобно представить в виде , тогда 

Найдем производную функции по формуле производной произведения

Вынесем за скобку . Получим

2. Приравняем производную к нулю:

. Первый множитель всегда больше нуля.

3.  Нанесем эти корни на координатную прямую и расставим знаки.

4. Производная меняет знак с “+” на “-” в точке х=0, следовательно это и есть точка максимума функции.

Ответ: 0

 

2. Задание B15 (№ 77435)

Найдите точку максимума функции .

1. 

2. 

3. Нанесем корни на числовую ось и расставим знаки.

Внимание! В этом месте школьники часто делают ошибку. Мы расставляем знаки для производной, которую получили в п.1. Иногда при нахождении корней производную умножают на (-1), и потом расставляют знаки уже для преобразованного уравнения производной. Это ошибка.

4. Производная меняет знак с “+” на “-” в точке х=2, следовательно это и есть точка максимума функции.

Ответ: 2

3. Задание B15 (№ 77454)

Найдите наименьшее значение функции     на отрезке .

1. 

2. 

Производная меняет знак в правом конце отрезка. При  производная функции  отрицательна, следовательно, функция убывает на отрезке . Следовательно, наименьшее значение она принимает в правом конце отрезка, то есть в точке х=9.

3. Подставим х=9 в уравнение функции:

Ответ: -8

4. Задание B15 (№ 77471)

Найдите точку максимума функции  .

ОДЗ функции .

1.

2. 

Нанесем корни на числовую ось и расставим знаки.   при . 

Производная меняет знак с “+” на “-” в точке х=-4, это и есть точка максимума функции .

Ответ: -4

И, наконец, предлагаю вам посмотреть ВИДЕОУРОК, в котором я показываю прием, позволяющий устно решать некоторые виды задач из задания В15.

Вероятно, Ваш браузер не поддерживается. Попробуйте скачать
Firefox

И.В. Фельдман, репетитор по математике.

Как определить знак производной по графику функции. Что такое производная

Производная функции – одна из сложных тем в школьной программе. Не каждый выпускник ответит на вопрос, что такое производная.

В этой статье просто и понятно рассказано о том, что такое производная и для чего она нужна . Мы не будем сейчас стремиться к математической строгости изложения. Самое главное – понять смысл.

Запомним определение:

Производная – это скорость изменения функции.

На рисунке – графики трех функций. Как вы думаете, какая из них быстрее растет?

Ответ очевиден – третья. У нее самая большая скорость изменения, то есть самая большая производная.

Вот другой пример.

Костя, Гриша и Матвей одновременно устроились на работу. Посмотрим, как менялся их доход в течение года:

На графике сразу все видно, не правда ли? Доход Кости за полгода вырос больше чем в два раза. И у Гриши доход тоже вырос, но совсем чуть-чуть. А доход Матвея уменьшился до нуля. Стартовые условия одинаковые, а скорость изменения функции, то есть производная , – разная. Что касается Матвея – у его дохода производная вообще отрицательна.

Интуитивно мы без труда оцениваем скорость изменения функции. Но как же это делаем?

На самом деле мы смотрим, насколько круто идет вверх (или вниз) график функции. Другими словами – насколько быстро меняется у с изменением х. Очевидно, что одна и та же функция в разных точках может иметь разное значение производной – то есть может меняться быстрее или медленнее.

Производная функции обозначается .

Покажем, как найти с помощью графика.

Нарисован график некоторой функции . Возьмем на нем точку с абсциссой . Проведём в этой точке касательную к графику функции. Мы хотим оценить, насколько круто вверх идет график функции. Удобная величина для этого – тангенс угла наклона касательной .

Производная функции в точке равна тангенсу угла наклона касательной, проведённой к графику функции в этой точке.

Обратите внимание – в качестве угла наклона касательной мы берем угол между касательной и положительным направлением оси .

Иногда учащиеся спрашивают, что такое касательная к графику функции. Это прямая, имеющая на данном участке единственную общую точку с графиком, причем так, как показано на нашем рисунке. Похоже на касательную к окружности.

Найдем . Мы помним, что тангенс острого угла в прямоугольном треугольнике равен отношению противолежащего катета к прилежащему. Из треугольника :

Мы нашли производную с помощью графика, даже не зная формулу функции. Такие задачи часто встречаются в ЕГЭ по математике под номером .

Есть и другое важное соотношение. Вспомним, что прямая задается уравнением

Величина в этом уравнении называется угловым коэффициентом прямой . Она равна тангенсу угла наклона прямой к оси .

.

Мы получаем, что

Запомним эту формулу. Она выражает геометрический смысл производной.

Производная функции в точке равна угловому коэффициенту касательной, проведенной к графику функции в этой точке.

Другими словами, производная равна тангенсу угла наклона касательной.

Мы уже сказали, что у одной и той же функции в разных точках может быть разная производная.

Посмотрим, как же связана производная с поведением функции.

Нарисуем график некоторой функции . Пусть на одних участках эта функция возрастает, на других – убывает, причем с разной скоростью. И пусть у этой функции будут точки максимума и минимума.

В точке функция возрастает. Касательная к графику, проведенная в точке , образует острый угол ; с положительным направлением оси . Значит, в точке производная положительна.

В точке наша функция убывает. Касательная в этой точке образует тупой угол ; с положительным направлением оси . Поскольку тангенс тупого угла отрицателен, в точке производная отрицательна.

Вот что получается:

Если функция возрастает, ее производная положительна.

Если убывает, ее производная отрицательна.

А что же будет в точках максимума и минимума? Мы видим, что в точках (точка максимума) и (точка минимума) касательная горизонтальна. Следовательно, тангенс угла наклона касательной в этих точках равен нулю, и производная тоже равна нулю.

Точка – точка максимума. В этой точке возрастание функции сменяется убыванием. Следовательно, знак производной меняется в точке с «плюса» на «минус».

В точке – точке минимума – производная тоже равна нулю, но ее знак меняется с «минуса» на «плюс».

Вывод: с помощью производной можно узнать о поведении функции всё, что нас интересует.

Если производная положительна, то функция возрастает.

Если производная отрицательная, то функция убывает.

В точке максимума производная равна нулю и меняет знак с «плюса» на «минус».

В точке минимума производная тоже равна нулю и меняет знак с «минуса» на «плюс».

Запишем эти выводы в виде таблицы:

возрастаетточка максимумаубываетточка минимумавозрастает
+00+

Сделаем два небольших уточнения. Одно из них понадобится вам при решении задачи . Другое – на первом курсе, при более серьезном изучении функций и производных.

Возможен случай, когда производная функции в какой-либо точке равна нулю, но ни максимума, ни минимума у функции в этой точке нет. Это так называемая :

В точке касательная к графику горизонтальна, и производная равна нулю. Однако до точки функция возрастала – и после точки продолжает возрастать. Знак производной не меняется – она как была положительной, так и осталась.

Бывает и так, что в точке максимума или минимума производная не существует. На графике это соответствует резкому излому, когда касательную в данной точке провести невозможно.

А как найти производную, если функция задана не графиком, а формулой? В этом случае применяется

{\large\bf Производная функции}

Рассмотрим функцию y=f(x) , заданную на интервале (a, b) . Пусть x – любое фиксированная точка интервала (a, b) , а Δx – произвольное число, такое, что значение x+Δx также принадлежит интервалу (a, b) . Это число Δx называют приращением аргумента.

Определение . Приращением функции y=f(x) в точке x , соответствующим приращению аргумента Δx , назовем число

Δy = f(x+Δx) – f(x) .

Считаем, что Δx ≠ 0 . Рассмотрим в данной фиксированной точке x отношение приращения функции в этой точке к соответствующему приращению аргумента Δx

Это отношение будем называть разностным отношением. Так как значение x мы считаем фиксированным, разностное отношение представляет собой функцию аргумента Δx . Эта функция определена для всех значений аргумента Δx , принадлежащих некоторой достаточно малой окрестности точки Δx=0 , за исключением самой точки Δx=0 . Таким образом, мы имеем право рассматривать вопрос о существовании предела указанной функции при Δx → 0 .

Определение . Производной функции y=f(x) в данной фиксированной точке x называется предел при Δx → 0 разностного отношения, то есть

При условии, что этот предел существует.

Обозначение . y′(x) или f′(x) .

Геометрический смысл производной : Производная от функции f(x) в данной точке x равна тангенсу угла между осью Ox и касательной к графику этой функции в соответствующей точке:

f′(x 0) = \tgα .

Механический смысл производной : Производная от пути по времени равна скорости прямолинейного движения точки:

Уравнение касательной к линии y=f(x) в точке M 0 (x 0 ,y 0) принимает вид

y-y 0 = f′(x 0) (x-x 0) .

Нормалью к кривой в некоторой ее точке называется перпендикуляр к касательной в той же точке. Если f′(x 0)≠ 0 , то уравнение нормали к линии y=f(x) в точке M 0 (x 0 ,y 0) записывается так:

Понятие дифференцируемости функции

Пусть функция y=f(x) определена на некотором интервале (a, b) , x – некоторое фиксированное значение аргумента из этого интервала, Δx – любое приращение аргумента, такое, что значение аргумента x+Δx ∈ (a, b) .

Определение . Функция y=f(x) называется дифференцируемой в данной точке x , если приращение Δy этой функции в точке x , соответствующее приращению аргумента Δx , может быть представимо в виде

Δy = A Δx +αΔx ,

где A – некоторое число, не зависящее от Δx , а α – функция аргумента Δx , являющая бесконечно малой при Δx→ 0 .

Так как произведение двух бесконечно малых функций αΔx является бесконечно малой более высокого порядка, чем Δx (свойство 3 бесконечно малых функций), то можем записать:

Δy = A Δx +o(Δx) .

Теорема . Для того, чтобы функция y=f(x) являлась дифференцируемой в данной точке x , необходимо и достаточно, чтобы она имела в этой точке конечную производную. При этом A=f′(x) , то есть

Δy = f′(x) Δx +o(Δx) .

Операцию нахождения производной обычно называют дифференцированием.

Теорема . Если функция y=f(x) x , то она непрерывна в этой точке.

Замечание . Из непрерывности функции y=f(x) в данной точке x , вообще говоря, не вытекает дифференцируемость функции f(x) в этой точке. Например, функция y=|x| – непрерывна в точке x=0 , но не имеет производной.

Понятие дифференциала функции

Определение . Дифференциалом функции y=f(x) называется произведение производной этой функции на приращение независимой переменной x :

dy = y′ Δx, df(x) = f′(x) Δx .

Для функции y=x получаем dy=dx=x′Δx = 1· Δx= Δx , то есть dx=Δx – дифференциал независимой переменной равен приращению этой переменной.

Таким образом, можем записать

dy = y′ dx, df(x) = f′(x) dx

Дифференциал dy и приращение Δy функции y=f(x) в данной точке x , оба отвечающие одному и тому же приращению аргумента Δx , вообще говоря, не равны друг другу.

Геометрический смысл дифференциала : Дифференциал функции равен приращению ординаты касательной к графику данной функции, когда аргумент получает приращение Δx .

Правила дифференцирования

Теорема . Если каждая из функций u(x) и v(x) дифференцируема в данной точке x , то сумма, разность, произведение и частное этих функций (частное при условии, что v(x)≠ 0 ) также дифференцируемы в этой точке, причем имеют место формулы:

Рассмотрим сложную функцию y=f(φ(x))≡ F(x) , где y=f(u) , u=φ(x) . В этом случае u называют промежуточным аргументом , x независимой переменной .

Теорема . Если y=f(u) и u=φ(x) – дифференцируемые функции своих аргументов, то производная сложной функции y=f(φ(x)) существует и равна произведению этой функции по промежуточному аргументу на производную промежуточного аргумента по независимой переменной, т. е.

Замечание . Для сложной функции, являющейся суперпозицией трех функций y=F(f(φ(x))) , правило дифференцирования имеет вид

y′ x = y′ u u′ v v′ x ,

где функции v=φ(x) , u=f(v) и y=F(u) – дифференцируемые функции своих аргументов.

Теорема . Пусть функция y=f(x) возрастает (или убывает) и непрерывна в некоторой окрестности точки x 0 . Пусть, кроме того, эта функция дифференцируема в указанной точке x 0 и ее производная в этой точке f′(x 0) ≠ 0 . Тогда в некоторой окрестности соответствующей точки y 0 =f(x 0) определена обратная для y=f(x) функция x=f -1 (y) , причем указанная обратная функция дифференцируема в соответствующей точке y 0 =f(x 0) и для ее производной в этой точке y справедлива формула

Таблица производных

Инвариантность формы первого дифференциала

Рассмотрим дифференциал сложной функции. Если y=f(x) , x=φ(t) – дифференцируемы функции своих аргументов, то производная функции y=f(φ(t)) выражается формулой

y′ t = y′ x x′ t .

По определению dy=y′ t dt , тогда получим

dy = y′ t dt = y′ x · x′ t dt = y′ x (x′ t dt) = y′ x dx ,

dy = y′ x dx .

Итак, доказали,

Свойство инвариантности формы первого дифференциала функции : как в случае, когда аргумент x является независимой переменной, так и в случае, когда аргумент x сам является дифференцируемой функцией новой переменной, дифференциал dy функции y=f(x) равен производной этой функции, умноженной на дифференциал аргумента dx .

Применение дифференциала в приближенных вычислениях

Мы показали, что дифференциал dy функции y=f(x) , вообще говоря, не равен приращению Δy этой функции. Тем не менее с точностью до бесконечно малой функции более высокого порядка малости, чем Δx , справедливо приближенное равенство

Δy ≈ dy .

Отношение называют относительной погрешностью равенства этого равенства. Так как Δy-dy=o(Δx) , то относительная погрешность данного равенства становится как угодно малой при уменьшении |Δх| .

Учитывая, что Δy=f(x+δ x)-f(x) , dy=f′(x)Δx , получим f(x+δ x)-f(x) ≈ f′(x)Δx или

f(x+δ x) ≈ f(x) + f′(x)Δx .

Это приближенное равенство позволяет с ошибкой o(Δx) заменить функцию f(x) в малой окрестности точки x (т.е. для малых значений Δx ) линейной функцией аргумента Δx , стоящей в правой части.

Производные высших порядков

Определение . Второй производной (или производной второго порядка) функции y=f(x) называется производная от ее первой производной.

Обозначение второй производной функции y=f(x) :

Механический смысл второй производной . Если функция y=f(x) описывает закон движения материальной точки по прямой линии, то вторая производная f″(x) равна ускорению движущейся точки в момент времени x .

Аналогично определяется третья, четвертая производная.

Определение . n -й производной (или производной n -го порядка) функции y=f(x) называется производная от ее n-1 -й производной:

y (n) =(y (n-1))′, f (n) (x)=(f (n-1) (x))′ .

Обозначения: y″′ , y IV , y V и т.д.

При решении различных задач геометрии, механики, физики и других отраслей знания возникла необходимость с помощью одного и того же аналитического процесса из данной функции y=f(x) получать новую функцию, которую называют производной функцией (или просто производной) данной функции f(x) и обозначают символом

Тот процесс, с помощью которого из данной функции f(x) получают новую функцию f ” (x) , называют дифференцированием и состоит он из следующих трех шагов: 1) даем аргументу x приращение  x и определяем соответствующее приращение функции  y = f(x+  x) -f(x) ; 2) составляем отношение

3) считая x постоянным, а  x 0, находим
, который обозначаем черезf ” (x) , как бы подчеркивая тем самым, что полученная функция зависит лишь от того значения x , при котором мы переходим к пределу. Определение : Производной y ” =f ” (x) данной функции y=f(x) при данном x называется предел отношения приращения функции к приращению аргумента при условии, что приращение аргумента стремится к нулю, если, конечно, этот предел существует, т.е. конечен. Таким образом,
, или

Заметим, что если при некотором значении x , например при x=a , отношение
при x 0 не стремится к конечному пределу, то в этом случае говорят, что функция f(x) при x=a (или в точке x=a ) не имеет производной или не дифференцируема в точке x=a .

2. Геометрический смысл производной.

Рассмотрим график функции у = f (х), дифференцируемой в окрест­ностях точки x 0

f(x)

Рассмотрим произвольную прямую, проходящую через точку гра­фика функции – точку А(x 0 , f (х 0)) и пересекающую график в некоторой точке B(x;f(x)). Такая прямая (АВ) называется секущей. Из ∆АВС: АС = ∆x; ВС =∆у; tgβ=∆y/∆x .

Так как АС || Ox, то ALO = BAC = β (как соответственные при параллельных). Но ALO – это угол наклона секущей АВ к положи­тельному направлению оси Ох. Значит, tgβ = k – угловой коэффициент прямой АВ.

Теперь будем уменьшать ∆х, т.е. ∆х→ 0. При этом точка В будет прибли­жаться к точке А по графику, а секущая АВ будет поворачиваться. Пре­дельным положением секущей АВ при ∆х→ 0 будет прямая (a), называемая касательной к графику функции у = f (х) в точке А.

Если перейти к пределу при ∆х → 0 в равенстве tgβ =∆y/∆x, то получим
илиtg =f “(x 0), так как
-угол накло­на касательной к положительному направлению оси Ох
, по определению производной. Но tg = k – угловой коэффициент каса­тельной, значит, k = tg = f “(x 0).

Итак, геометрический смысл производной заключается в следую­щем:

Производная функции в точке x 0 равна угловому коэффициенту ка­сательной к графику функции, проведенной в точке с абсциссой x 0 .

3.

Физический смысл производной.

Рассмотрим движение точки по прямой. Пусть задана координата точки в любой момент времени x(t). Известно (из курса физики), что средняя скорость за промежуток времени равна отношению расстояния, пройденного за этот промежуток времени, на время, т.е.

Vср = ∆x/∆t. Перейдем к пределу в последнем равенстве при ∆t → 0.

lim Vср (t) = (t 0) – мгновенная скорость в момент времени t 0 , ∆t → 0.

а lim = ∆x/∆t = x”(t 0) (по определению производной).

Итак, (t) =x”(t).

Физический смысл производной заключается в следующем: произ­водная функции y = f (x ) в точке x 0 – это скорость изменения функции f (х) в точке x 0

Производная применяется в физике для нахождения скорости по известной функции координаты от времени, ускорения по известной функции скорости от времени.

(t) = x”(t) – скорость,

a(f) = ”(t) – ускорение, или

Если известен закон движения материальной точки по окружности, то можно найти угловую скорость и угловое ускорение при вращатель­ном движении:

φ = φ(t) – изменение угла от времени,

ω = φ”(t) – угловая скорость,

ε = φ”(t) – угловое ускорение, или ε = φ”(t).

Если известен закон распределения массы неоднородного стержня, то можно найти линейную плотность неоднородного стержня:

m = m(х) – масса,

x  , l – длина стержня,

р = m”(х) – линейная плотность.

С помощью производной решаются задачи из теории упругости и гармонических колебаний. Так, по закону Гука

F = -kx, x – переменная координата, k- коэффициент упругости пружины. Положив ω 2 =k/m, получим дифференциальное уравнение пружинного маятника х”(t) + ω 2 x(t) = 0,

где ω = √k/√m частота колебаний (l/c), k – жесткость пружины (H/m).

Уравнение вида у” + ω 2 y = 0 называется уравнением гармонических колебаний (механических, электрических, электромагнитных). Решени­ем таких уравнений является функция

у = Asin(ωt + φ 0) или у = Acos(ωt + φ 0), где

А – амплитуда колебаний, ω – циклическая частота,

φ 0 – начальная фаза.

Содержание статьи

ПРОИЗВОДНАЯ –производной функции y = f (x ), заданной на некотором интервале (a , b ) в точке x этого интервала, называется предел, к которому стремится отношение приращения функции f в этой точке к соответствующему приращению аргумента, когда приращение аргумента стремится к нулю.

Производную принято обозначать так:

Широко употребляются и другие обозначения:

Мгновенная скорость.

Пусть точка M движется по прямой. Расстояние s движущейся точки, отсчитываемое от некоторого начального ее положения M 0 , зависит от времени t , т.е. s есть функция времени t : s = f (t ). Пусть в некоторый момент времени t движущаяся точка M находилась на расстоянии s от начального положения M 0, а в некоторый следующий момент t + Dt оказалась в положении M 1 – на расстоянии s + Ds от начального положения (см. рис .).

Таким образом, за промежуток времени Dt расстояние s изменилось на величину Ds . В этом случае говорят, что за промежуток времени Dt величина s получила приращение Ds .

Средняя скорость не может во всех случаях точно охарактеризовать быстроту перемещения точки M в момент времени t . Если, например, тело в начале промежутка Dt перемещалось очень быстро, а в конце очень медленно, то средняя скорость не сможет отразить указанных особенностей движения точки и дать представление об истинной скорости ее движения в момент t . Чтобы точнее выразить истинную скорость с помощью средней скорости, надо взять меньший промежуток времени Dt . Наиболее полно характеризует скорость движения точки в момент t тот предел, к которому стремится средняя скорость при Dt ® 0. Этот предел называют скоростью движения в данный момент:

Таким образом, скоростью движения в данный момент называется предел отношения приращения пути Ds к приращению времени Dt , когда приращение времени стремится к нулю. Так как

Геометрическое значение производной. Касательная к графику функции.

Построение касательных – одна из тех задач, которые привели к рождению дифференциального исчисления. Первый опубликованный труд, относящийся к дифференциальному исчислению и принадлежащий перу Лейбница, имел название Новый метод максимумов и минимумов, а также касательных, для которого не служат препятствием ни дробные, ни иррациональные величины, и особый для этого род исчисления .

Пусть кривая есть график функции y = f (x ) в прямоугольной системе координат (см . рис.).

При некотором значении x функция имеет значение y = f (x ). Этим значениям x и y на кривой соответствует точка M 0(x , y ). Если аргументу x дать приращение Dx , то новому значению аргумента x + Dx соответствует новое значение функции y+ Dy = f (x + Dx ). Соответствующей ему точкой кривой будет точка M 1(x + Dx , y + Dy ). Если провести секущую M 0M 1 и обозначить через j угол, образованный секущей с положительным направлением оси Ox , из рисунка непосредственно видно, что .

Если теперь Dx стремится к нулю, то точка M 1 перемещается вдоль кривой, приближаясь к точке M 0, и угол j изменяется с изменением Dx . При Dx ® 0 угол j стремится к некоторому пределу a и прямая, проходящая через точку M 0 и составляющая с положительным направлением оси абсцисс угол a, будет искомой касательной. Ее угловой коэффициент:

Следовательно, f ´(x ) = tga

т.е. значение производной f ´(x ) при данном значении аргумента x равняется тангенсу угла, образованного касательной к графику функции f (x ) в соответствующей точке M 0(x ,y ) с положительным направлением оси Ox .

Дифференцируемость функций.

Определение. Если функция y = f (x ) имеет производную в точке x = x 0, то функция дифференцируема в этой точке.

Непрерывность функции, имеющей производную. Теорема.

Если функция y = f (x ) дифференцируема в некоторой точке x = x 0, то она в этой точке непрерывна.

Таким образом, в точках разрыва функция не может иметь производной. Обратное заключение неверно, т.е. из того, что в какой-нибудь точке x = x 0 функция y = f (x ) непрерывна не следует, что она в этой точке дифференцируема. Например, функция y = |x | непрерывна для всех x (–Ґ х x = 0 не имеет производной. В этой точке не существует касательной к графику. Есть правая касательная и левая, но они не совпадают.

Некоторые теоремы о дифференцируемых функциях. Теорема о корнях производной (теорема Ролля). Если функция f (x ) непрерывна на отрезке [a ,b ], дифференцируема во всех внутренних точках этого отрезка и на концах x = a и x = b обращается в нуль (f (a ) = f (b ) = 0), то внутри отрезка [a ,b ] существует, по крайней мере одна, точка x = с , a c b, в которой производная f ў(x ) обращается в нуль, т.е. f ў(c ) = 0.

Теорема о конечных приращениях (теорема Лагранжа). Если функция f (x ) непрерывна на отрезке [a , b ] и дифференцируема во всех внутренних точках этого отрезка, то внутри отрезка [a , b ] найдется по крайней мере одна точка с , a c b, что

f (b ) – f (a ) = f ў(c )(b a ).

Теорема об отношении приращений двух функций (теорема Коши). Если f (x ) и g (x ) – две функции, непрерывные на отрезке [a , b ] и дифференцируемые во всех внутренних точках этого отрезка, причем g ў(x ) нигде внутри этого отрезка не обращается в нуль, то внутри отрезка [a , b ] найдется такая точка x = с , a c b, что

Производные различных порядков.

Пусть функция y = f (x ) дифференцируема на некотором отрезке [a , b ]. Значения производной f ў(x ), вообще говоря, зависят от x , т. е. производная f ў(x ) представляет собой тоже функцию от x . При дифференцировании этой функции получается так называемая вторая производная от функции f (x ), которая обозначается f ўў (x ).

Производной n- го порядка от функции f (x ) называется производная (первого порядка) от производной n- 1го и обозначается символом y (n ) = (y (n – 1))ў.

Дифференциалы различных порядков.

Дифференциал функции y = f (x ), где x – независимая переменная, есть dy = f ў(x )dx , некоторая функция от x , но от x может зависеть только первый сомножитель f ў(x ), второй же сомножитель (dx ) является приращением независимой переменной x и от значения этой переменной не зависит. Так как dy есть функция от x , то можно определить дифференциал этой функции. Дифференциал от дифференциала функции называется вторым дифференциалом или дифференциалом второго порядка этой функции и обозначается d 2y :

d (dx ) = d 2y = f ўў(x )(dx ) 2 .

Дифференциалом n- го порядка называется первый дифференциал от дифференциала n- 1го порядка:

d n y = d (d n –1 y ) = f (n )(x )dx (n ).

Частная производная.

Если функция зависит не от одного, а от нескольких аргументов x i (i изменяется от 1 до n , i = 1, 2,… n ), f (x 1, x 2,… x n ), то в дифференциальном исчислении вводится понятие частной производной, которая характеризует скорость изменения функции нескольких переменных, когда изменяется только один аргумент, например, x i . Частная производная 1-ого порядка по x i определяется как обычная производная, при этом предполагается, что все аргументы, кроме x i , сохраняют постоянные значения. Для частных производных вводятся обозначения

Определенные таким образом частные производные 1-ого порядка (как функции тех же аргументов) могут, в свою очередь, также иметь частные производные, это частные производные второго порядка и т.д. Взятые по разным аргументам такие производные называются смешанными. Непрерывные смешанные производные одного порядка не зависят от порядка дифференцирования и равны между собой.

Анна Чугайнова


Дата: 20.11.2014

Таблица производных.

Производная – одно из главных понятий высшей математики. В этом уроке мы познакомимся с этим понятием. Именно познакомимся, без строгих математических формулировок и доказательств.

Это знакомство позволит:

Понимать суть несложных заданий с производной;

Успешно решать эти самые несложные задания;

Подготовиться к более серьёзным урокам по производной.

Сначала – приятный сюрприз.)

Строгое определение производной основано на теории пределов и штука достаточно сложная. Это огорчает. Но практическое применение производной, как правило, не требует таких обширных и глубоких знаний!

Для успешного выполнения большинства заданий в школе и ВУЗе достаточно знать всего несколько терминов – чтобы понять задание, и всего несколько правил – чтобы его решить. И всё. Это радует.

Приступим к знакомству?)

Термины и обозначения.

В элементарной математике много всяких математических операций. Сложение, вычитание умножение, возведение в степень, логарифмирование и т.д. Если к этим операциям добавить ещё одну, элементарная математика становится высшей. Эта новая операция называется дифференцирование. Определение и смысл этой операции будут рассмотрены в отдельных уроках.

Здесь же важно понять, что дифференцирование – это просто математическая операция над функцией. Берём любую функцию и, по определённым правилам, преобразовываем её. В результате получится новая функция. Вот эта новая функция и называется: производная.

Дифференцирование – действие над функцией.

Производная – результат этого действия.

Так же, как, например, сумма – результат сложения. Или частное – результат деления.

Зная термины, можно, как минимум, понимать задания.) Формулировки бывают такие: найти производную функции; взять производную; продифференцировать функцию; вычислить производную и т.п. Это всё одно и то же. Разумеется, бывают и более сложные задания, где нахождение производной (дифференцирование) будет всего лишь одним из шагов решения задания.

Обозначается производная с помощью штришка вверху справа над функцией. Вот так: y” или f”(x) или S”(t) и так далее.

Читается игрек штрих, эф штрих от икс, эс штрих от тэ, ну вы поняли…)

Штрих также может обозначать производную конкретной функции, например: (2х+3)” , (x 3 )” , (sinx)” и т.д. Часто производная обозначается с помощью дифференциалов, но такое обозначение в этом уроке мы рассматривать не будем.

Предположим, что понимать задания мы научились. Осталось всего ничего – научиться их решать.) Напомню ещё раз: нахождение производной – это преобразование функции по определённым правилам. Этих правил, на удивление, совсем немного.

Чтобы найти производную функции, надо знать всего три вещи. Три кита, на которых стоит всё дифференцирование. Вот они эти три кита:

1. Таблица производных (формулы дифференцирования).

3. Производная сложной функции.

Начнём по порядку. В этом уроке рассмотрим таблицу производных.

Таблица производных.

В мире – бесконечное множество функций. Среди этого множества есть функции, которые наиболее важны для практического применения. Эти функции сидят во всех законах природы. Из этих функций, как из кирпичиков, можно сконструировать все остальные. Этот класс функций называется элементарные функции. Именно эти функции и изучаются в школе – линейная, квадратичная, гипербола и т.п.

Дифференцирование функций “с нуля”, т. е. исходя из определения производной и теории пределов – штука достаточно трудоёмкая. А математики – тоже люди, да-да!) Вот и упростили себе (и нам) жизнь. Они вычислили производные элементарных функций до нас. Получилась таблица производных, где всё уже готово.)

Вот она, эта табличка для самых популярных функций. Слева – элементарная функция, справа – её производная.

Рекомендую обратить внимание на третью группу функций в этой таблице производных. Производная степенной функции – одна из самых употребительных формул, если только не самая употребительная! Намёк понятен?) Да, таблицу производных желательно знать наизусть. Кстати, это не так трудно, как может показаться. Попробуйте решать побольше примеров, таблица сама и запомнится!)

Найти табличное значение производной, как вы понимаете, задание не самое трудное. Поэтому очень часто в подобных заданиях встречаются дополнительные фишки. Либо в формулировке задания, либо в исходной функции, которой в таблице – вроде и нету. ..

Рассмотрим несколько примеров:

1. Найти производную функции y = x 3

Такой функции в таблице нет. Но есть производная степенной функции в общем виде (третья группа). В нашем случае n=3. Вот и подставляем тройку вместо n и аккуратно записываем результат:

(x 3) ” = 3·x 3-1 = 3x 2

Вот и все дела.

Ответ: y” = 3x 2

2. Найти значение производной функции y = sinx в точке х = 0.

Это задание означает, что надо сначала найти производную от синуса, а затем подставить значение х = 0 в эту самую производную. Именно в таком порядке! А то, бывает, сразу подставляют ноль в исходную функцию… Нас же просят найти не значение исходной функции, а значение её производной. Производная, напомню – это уже новая функция.

По табличке находим синус и соответствующую производную:

y” = (sin x)” = cosx

Подставляем ноль в производную:

y”(0) = cos 0 = 1

Это и будет ответ.

3. Продифференцировать функцию:

Что, внушает?) Такой функции в таблице производных и близко нет.

Напомню, что продифференцировать функцию – это просто найти производную этой функции. Если забыть элементарную тригонометрию, искать производную нашей функции достаточно хлопотно. Таблица не помогает…

Но если увидеть, что наша функция – это косинус двойного угла , то всё сразу налаживается!

Да-да! Запомните, что преобразование исходной функции до дифференцирования вполне допускается! И, случается, здорово облегчает жизнь. По формуле косинуса двойного угла:

Т.е. наша хитрая функция есть не что иное, как y = cosx . А это – табличная функция. Сразу получаем:

Ответ: y” = – sin x .

Пример для продвинутых выпускников и студентов:

4. Найти производную функции:

Такой функции в таблице производных нет, разумеется. Но если вспомнить элементарную математику, действия со степенями. .. То вполне можно упростить эту функцию. Вот так:

А икс в степени одна десятая – это уже табличная функция! Третья группа, n=1/10. Прямо по формуле и записываем:

Вот и всё. Это будет ответ.

Надеюсь, что с первым китом дифференцирования – таблицей производных – всё ясно. Осталось разобраться с двумя оставшимися китами. В следующем уроке освоим правила дифференцирования.

Максимумы, минимумы и экстремумы функций

Минимумом называют точку на функции, в которой значение функции меньше, чем в соседних точках.

Максимумом называют точку на функции, в которой значение функции больше, чем в соседних точках.

Также можно сказать, что в этих точках меняется направление движения функции: если функция перестает падать и начинает расти – это точка минимума, наоборот – максимума.


Минимумы и максимумы вместе именуют

экстремумами функции.

Иными словами, все пять точек, выделенных на графике выше, являются экстремумами.


В точках экстремумов (т.е. максимумов и минимумов) производная равна нулю.

Благодаря этому найти эти точки не составляет проблем, даже если у вас нет графика функции.

Внимание! Когда пишут экстремумы или максимумы/минимумы имеют в виду значение функции т.е. \(y\). Когда пишут точки экстремумов или точки максимумов/минимумов имеют в виду иксы в которых достигаются максимумы/минимумы. Например, на рисунке выше, \(-5\) точка минимума (или точка экстремума), а \(1\) – минимум (или экстремум).

Как найти точки экстремумов функции по графику производной (7 задание ЕГЭ)?

Давайте вместе найдем количество точек экстремума функции по графику производной на примере:


У нас дан график производная — значит ищем в каких точках на графике производная равна нулю. Очевидно, это точки \(-13\), \(-11\), \(-9\),\(-7\) и \(3\). Количество точек экстремума функции – \(5\).

Внимание! Если дан график производной функции, а нужно найти точки экстремумов функции, мы не считаем максимумы и минимумы производной! Мы считаем точки, в которых производная функции обращается в ноль (т.е. пересекает ось \(x\)).

         


Как найти точки максимумов или минимумов функции по графику производной (7 задание ЕГЭ)?

Чтобы ответить на этот вопрос, нужно вспомнить еще два важных правил:

– Производная положительна там, где функция возрастает.


– Производная отрицательна там, где функция убывает.

С помощью этих правил давайте найдем на графике производной точки минимума и максимума функции.


Понятно, что минимумы и максимумы надо искать среди точек экстремумов, т.е. среди \(-13\), \(-11\), \(-9\),\(-7\) и \(3\).

Чтобы проще было решать задачу расставим на рисунке сначала знаки плюс и минус, обозначающие знак производной. Потом стрелки – обозначающие возрастание, убывания функции.


Начнем с \(-13\): до \(-13\) производная положительна т.е. функция растет, после – производная отрицательна т.е. функция падает. Если это представить, то становится ясно, что \(-13\) – точка максимума.

\(-11\): производная сначала положительна, а потом отрицательна, значит функция возрастает, а потом убывает. Опять попробуйте это мысленно нарисовать и вам станет очевидно, что \(-11\) – это минимум.

\(- 9\): функция возрастает, а потом убывает – максимум.

\(-7\): минимум.

\(3\): максимум.

Все вышесказанное можно обобщить следующими выводами:

– Функция имеет максимум там, где производная равна нулю и меняет знак с плюса на минус.


– Функция имеет минимум там, где производная равна нулю и меняет знак с минуса на плюс.

Как найти точки максимумов и минимумов если известна формула функции (12 задание ЕГЭ)?

Чтобы ответить на этот вопрос, нужно делать все то же, что и в предыдущем пункте: находить где производная положительна, где отрицательна и где равна нулю. Чтобы было понятнее напишу алгоритм с примером решения:

  1. Найдите производную функции \(f'(x)\). 
  2. Найдите корни уравнения \(f'(x)=0\). 
  3. Нарисуйте ось \(x\) и отметьте на ней точки полученные в пункте 2, изобразите дугами промежутки, на которые разбивается ось. Подпишите над осью \(f'(x)\), а под осью \(f(x)\).
  4. Определите знак производной в каждом промежутке (методом интервалов). 
  5. Поставьте знак производной в каждом промежутке (над осью), а стрелкой укажите возрастание (↗) или убывание (↘) функции (под осью). 
  6. Определите, как изменился знак производной при переходе через точки, полученные в пункте 2:
    – если \(f’(x)\) изменила знак с «\(+\)» на «\(-\)», то \(x_1\) – точка максимума;
    – если \(f’(x)\) изменила знак с «\(-\)» на «\(+\)», то \(x_3\) – точка минимума;
    – если \(f’(x)\) не изменила знак, то \(x_2\) – может быть точкой перегиба.2-4=0\)
                   \(x=±2\)

    3. – 6. Нанесем точки на числовую ось и определим, как меняется знак производной и как движется функция:


    Теперь очевидно, что точкой максимума является \(-2\).

    Ответ. \(-2\).

    Смотрите также:
    Связь функции и её производной | 7 задача ЕГЭ
    Разбор задач на поиск экстремумов, минимумов и максимумов


    Скачать статью

    Исследование функции с помощью производной /qualihelpy

    Рассмотрим функции  и , которые непрерывны на отрезке  и дифференцируемы на интервале .
    Теорема Ферма : если функция  в точке  имеет локальный экстремум, то  .
    Геометрический смысл теоремы: касательная к графику функции в точке  параллельна оси абсцисс. 

    Теорема Лагранжа:  , где .

    Геометрический смысл теоремы: касательная к графику функции в точке   параллельна секущей, соединяющей концы графика этой функции.

    Теорема Ролля: если  и  , то .

    Геометрический смысл теоремы: у графика функции существует точка, в которой касательная параллельна оси абсцисс.

    Теорема Коши: если  , то .

    Исследование функции с помощью первой производной

    С помощью производной функции можно определить характер монотонности функции, точки экстремума, а также ее наибольшее и наименьшее значение на заданном промежутке.

    Достаточное условие возрастания (убывания) функции:

    а) если на заданном промежутке   , то функция возрастает на этом промежутке;

    б) если   , то функция убывает на этом промежутке.

    Экстремум функции

    Максимумом (минимумом) функции   называют такое ее значение, которое больше (меньше) всех ее других значений в окрестности рассматриваемой точки.

    Максимум и минимум функции имеют локальный характер, поскольку отдельные минимумы некоторой функции могут оказаться больше максимумов той же функции (рис. 6.4).

    Максимум и минимум функции называются  экстремумом функции . Значение аргумента, при котором достигается экстремум, называется точкой экстремума . На рисунке 6.4 значения , , ,  и  являются точками экстремума рассматриваемой функции.

     

    Критическими точками функции называют те значения аргумента, при которых производная функции равна нулю или не существует. Критические точки функции находят, решая уравнение: .

    Алгоритм нахождения точек экстремума функции:

    1) находим область определения функции  ;
    2) находим ;

    3) находим критические точки функции, решая уравнение ;

    4) наносим критические точки на область определения функции;

    5) определяем знак производной функции на полученных промежутках;

    6) определяем точки экстремума функции по правилу: 
    если при переходе через критическую точку производная меняет знак c «+» на «–», то имеем точку максимума, а если с «–» на «+», то имеем точку минимума.

    Рассмотрим функцию   на отрезке . Свое наибольшее и наименьшее значение она может принимать либо на концах отрезка, либо в точках экстремума.

    Алгоритм нахождения наибольшего и наименьшего значений функции на заданном отрезке:  

    1) находим ;

    2) находим критические точки функции, решая уравнение ;

    3) находим значение функции на концах отрезка и в критических точках, принадлежащих данному отрезку;

    4) определяем наибольшее и наименьшее значение из полученных.

    Исследование функции с помощью второй производной

    Критическими точками второго рода функции  называют те значения аргумента, при которых вторая производная этой функции равна нулю или не существует.

    Критические точки второго рода функции находят, решая уравнение .

    Если при переходе через критическую точку второго рода вторая производная функции меняет знак, то имеем точку перегиба  графика функции.

    Если на некотором промежутке выполняется неравенство , то функция  вогнута на этом промежутке, а если , то функция выпукла на этом промежутке.

    Производная просто. Производная функции. Геометрический смысл производной

    Исследование функций. В этой статье мы поговорим о задачах, в которых рассматриваются функции и в условии стоят вопросы связанные с их исследованием. Рассмотрим основные теоретические моменты, которые необходимо знать и понимать для их решения.

    Это целая группа задач входящих в ЕГЭ по математике. Обычно ставится вопрос о нахождении точек максимума (минимума) или определения наибольшего (наименьшего) значения функции на заданном интервале. Рассматриваются:

    — Степенные и иррациональные функции.

    — Рациональные функции.

    — Исследование произведений и частных.

    — Логарифмические функции.

    — Тригонометрические функции.

    Если вы поняли теорию пределов, понятие производной, свойства производной для исследования графиков функций и её , то такие задачи никакого затруднения у вас не вызовут и вы решите их с лёгкостью.

    Информация ниже — это теоретические моменты, понимание которых позволит осознать, как решать подобные задачи. Постараюсь изложить их именно так, чтобы даже тот, кто эту тему пропустил или изучил слабо, смог без особых затруднений решать подобные задачи.

    В задачах данной группы, как уже сказано, требуется найти либо точку минимума (максимума) функции, либо наибольшее (наименьшее) значение функции на интервале.

    Точки минимума, максимума. Свойства производной.

    Рассмотрим график функции:


    Точка А – это точка максимума, на интервале от О до А функция возрастает, на интервале от А до В убывает.

    Точка В – это точка минимума, на интервале от А до В функция убывает, на интервале от В до С возрастает.

    В данных точках (А и В) производная обращается в нуль (равна нулю).

    Касательные в этих точках параллельны оси ox .

    Добавлю, что точки, в которых функция меняет своё поведение с возрастания на убывание (и наоборот, с убывания на возрастание), называются экстремумами.

    Важный момент:

    1. Производная на интервалах возрастания имеет положительный знак (п ри подстановке значения из интервала в производную получается положительное число).

    Значит, если производная в определённой точке из некоторого интервала имеет положительное значение, то график функции на этом интервале возрастает.

    2. На интервалах убывания производная имеет отрицательный знак (при подстановке значения из интервала в выражение производной получается отрицательное число).

    Значит, если производная в определённой точке из некоторого интервала имеет отрицательное значение, то график функции на этом интервале убывает.

    Это надо чётко уяснить!!!

    Таким образом, вычислив производную и приравняв её к нулю, можно найти точки, которые разбивают числовую ось на интервалы. На каждом из этих интервалов можно определить знак производной и далее сделать вывод о её возрастании или убывании.

    *Отдельно следует сказать о точках, в которых производая не существует. Например, можем получить производную, знаменатель которой при определённом х обращается в нуль. Понятно, что при таком х производная не существует. Так вот, данную точку также необходимо учитывать при определени интервалов возрастания (убывания).

    Функция в точках, где производная равна нулю меняет свой знак не всегда. Об этом будет отдельная статья. На самом ЕГЭ таких задач не будет.

    Вышеизложенные свойства необходимы для исследования поведения функции на возрастание и убывание.

    Что ещё необходимо знать для решения оговоренных задач: таблицу производных и правила дифференцирования. Без этого никак. Это базовые знания, в теме производной. Производные элементарных функций вы должны знать на отлично.

    Вычисляя производную сложной функции f (g (x )), представьте, что функция g (x ) это переменная и далее вычисляйте производную f ’(g (x )) по табличным формулам как обычную производную от переменной. Затем полученный результат умножьте на производную функции g (x ) .

    Посмотрите видеоурок Максима Семенихина о сложной функции:

    Задачи на нахождение точек максимума и минимума

    Алгоритм нахождения точек максимума (минимума) функции:

    1. Находим производную функции f ’(x ).

    2. Находим нули производной (приравниванием производную к нулю f ’(x )=0 и решаем полученное уравнение). Также находим точки в которых производная не существует (в частности это касается дробно-рациональных функций).

    3. Отмечаем полученные значения на числовой прямой и определяем знаки производной на этих интервалах путём подстановки значений из интервалов в выражение производной.

    Вывод будет один из двух:

    1. Точка максимума это точка, в которой производная меняет значение с положительного на отрицательное.

    2. Точка минимума это точка, в которой производная меняет значение с отрицательного на положительное.

    Задачи на нахождение наибольшего или наименьшего значения

    функции на интервале.

    В другом типе задач требуется найти наибольшее или наименьшее значение функции на заданном интервале.

    Алгоритм нахождения наибольшего (наименьшего) значения функции:

    1. Определяем, есть ли точки максимума (минимума). Для этого находим производную f ’(x ) , затем решаем f ’(x )=0 (пункты 1 и 2 из предыдущего алгоритма).

    2. Определяем, принадлежат ли полученные точки заданному интервалу и записываем лежащие в его пределах.

    3. Подставляем в исходную функцию (не в производную, а в данную в условии) границы данного интервала и точки (максимума-минимума), лежащие в пределах интервала (п.2).

    4. Вычисляем значения функции.

    5. Выбираем из полученных наибольшее (наименьше) значение, в зависимости от того, какой вопрос был поставлен в задаче и далее записываем ответ.

    Вопрос: для чего в задачах на нахождение наибольшего (наименьшего) значения функции необходимо искать точки максимума (минимума)?

    Ответ лучше всего это проиллюстрировать, посмотрите схематичное изображение графиков, задаваемых функций:



    В случаях 1 и 2 достаточно подставить границы интервала, чтобы определить наибольшее или наименьшее значение функции. В случаях 3 и 4 необходимо найти нули функции (точки максимума-минимума). Если мы подставим границы интервала (не находя нули функции), то получим неверный ответ, это видно по графикам.

    И всё дело в том, что мы по заданной функции не можем увидеть как выглядит график на интервале (имеет ли он максимум или минимум в пределах интервала). Потому находите нули функции обязательно!!!

    Если уравнение f’(x )=0 не будет иметь решения, это значит, что точек максимума-минимума нет (рисунок 1,2), и для нахождения поставленной задачи в данную функцию подставляем только границы интервала.

    Ещё один важный момент. Помните, что ответом должно быть целое число или конечная десятичная дробь. При вычислении наибольшего и наименьшего значения функции вы будете получать выражения с числом е и Пи, а также выражения с корнем. Запомните, что до конца вам их вычислять не нужно, и так понятно, что результат таких выражений ответом являться не будет. Если возникнет желание вычислить такое значение, то сделайте это (числа: е ≈ 2,71 Пи ≈ 3,14).

    Много написал, запутал наверное? По конкретным примерам вы увидите, что всё просто.

    Далее хочу открыть вам маленький секрет. Дело в том, что многие задания можно решить без знания свойств производной и даже без правил дифференцирования. Об этих нюансах я вам обязательно расскажу и покажу как это делается? не пропустите!

    Но тогда зачем же я вообще изложил теорию и ещё сказал, что её нужно знать обязательно. Всё верно – знать надо. Если её поймёте, тогда никакая задача в этой теме в тупик вас не поставит.

    Те «хитрости», о которых вы узнаете, помогут вам при решении конкретных (некоторых) прототипов задач. К ак дополнительный инструмент эти приёмы использовать, конечно, удобно. Задачу можно решить в 2-3 раза быстрее и сэкономить время на решение части С.

    Всего доброго!

    С уважением, Александр Крутицких.

    P.S: Буду благодарен Вам, если расскажите о сайте в социальных сетях.

    Содержание статьи

    ПРОИЗВОДНАЯ –производной функции y = f (x ), заданной на некотором интервале (a , b ) в точке x этого интервала, называется предел, к которому стремится отношение приращения функции f в этой точке к соответствующему приращению аргумента, когда приращение аргумента стремится к нулю.

    Производную принято обозначать так:

    Широко употребляются и другие обозначения:

    Мгновенная скорость.

    Пусть точка M движется по прямой. Расстояние s движущейся точки, отсчитываемое от некоторого начального ее положения M 0 , зависит от времени t , т.е. s есть функция времени t : s = f (t ). Пусть в некоторый момент времени t движущаяся точка M находилась на расстоянии s от начального положения M 0, а в некоторый следующий момент t + Dt оказалась в положении M 1 – на расстоянии s + Ds от начального положения (см. рис .).

    Таким образом, за промежуток времени Dt расстояние s изменилось на величину Ds . В этом случае говорят, что за промежуток времени Dt величина s получила приращение Ds .

    Средняя скорость не может во всех случаях точно охарактеризовать быстроту перемещения точки M в момент времени t . Если, например, тело в начале промежутка Dt перемещалось очень быстро, а в конце очень медленно, то средняя скорость не сможет отразить указанных особенностей движения точки и дать представление об истинной скорости ее движения в момент t . Чтобы точнее выразить истинную скорость с помощью средней скорости, надо взять меньший промежуток времени Dt . Наиболее полно характеризует скорость движения точки в момент t тот предел, к которому стремится средняя скорость при Dt ® 0. Этот предел называют скоростью движения в данный момент:

    Таким образом, скоростью движения в данный момент называется предел отношения приращения пути Ds к приращению времени Dt , когда приращение времени стремится к нулю. Так как

    Геометрическое значение производной. Касательная к графику функции.

    Построение касательных – одна из тех задач, которые привели к рождению дифференциального исчисления. Первый опубликованный труд, относящийся к дифференциальному исчислению и принадлежащий перу Лейбница, имел название Новый метод максимумов и минимумов, а также касательных, для которого не служат препятствием ни дробные, ни иррациональные величины, и особый для этого род исчисления .

    Пусть кривая есть график функции y = f (x ) в прямоугольной системе координат (см . рис.).

    При некотором значении x функция имеет значение y = f (x ). Этим значениям x и y на кривой соответствует точка M 0(x , y ). Если аргументу x дать приращение Dx , то новому значению аргумента x + Dx соответствует новое значение функции y+ Dy = f (x + Dx ). Соответствующей ему точкой кривой будет точка M 1(x + Dx , y + Dy ). Если провести секущую M 0M 1 и обозначить через j угол, образованный секущей с положительным направлением оси Ox , из рисунка непосредственно видно, что .

    Если теперь Dx стремится к нулю, то точка M 1 перемещается вдоль кривой, приближаясь к точке M 0, и угол j изменяется с изменением Dx . При Dx ® 0 угол j стремится к некоторому пределу a и прямая, проходящая через точку M 0 и составляющая с положительным направлением оси абсцисс угол a, будет искомой касательной. Ее угловой коэффициент:

    Следовательно, f ´(x ) = tga

    т.е. значение производной f ´(x ) при данном значении аргумента x равняется тангенсу угла, образованного касательной к графику функции f (x ) в соответствующей точке M 0(x ,y ) с положительным направлением оси Ox .

    Дифференцируемость функций.

    Определение. Если функция y = f (x ) имеет производную в точке x = x 0, то функция дифференцируема в этой точке.

    Непрерывность функции, имеющей производную. Теорема.

    Если функция y = f (x ) дифференцируема в некоторой точке x = x 0, то она в этой точке непрерывна.

    Таким образом, в точках разрыва функция не может иметь производной. Обратное заключение неверно, т.е. из того, что в какой-нибудь точке x = x 0 функция y = f (x ) непрерывна не следует, что она в этой точке дифференцируема. Например, функция y = |x | непрерывна для всех x (–Ґ х x = 0 не имеет производной. В этой точке не существует касательной к графику. Есть правая касательная и левая, но они не совпадают.

    Некоторые теоремы о дифференцируемых функциях. Теорема о корнях производной (теорема Ролля). Если функция f (x ) непрерывна на отрезке [a ,b ], дифференцируема во всех внутренних точках этого отрезка и на концах x = a и x = b обращается в нуль (f (a ) = f (b ) = 0), то внутри отрезка [a ,b ] существует, по крайней мере одна, точка x = с , a c b, в которой производная f ў(x ) обращается в нуль, т.е. f ў(c ) = 0.

    Теорема о конечных приращениях (теорема Лагранжа). Если функция f (x ) непрерывна на отрезке [a , b ] и дифференцируема во всех внутренних точках этого отрезка, то внутри отрезка [a , b ] найдется по крайней мере одна точка с , a c b, что

    f (b ) – f (a ) = f ў(c )(b a ).

    Теорема об отношении приращений двух функций (теорема Коши). Если f (x ) и g (x ) – две функции, непрерывные на отрезке [a , b ] и дифференцируемые во всех внутренних точках этого отрезка, причем g ў(x ) нигде внутри этого отрезка не обращается в нуль, то внутри отрезка [a , b ] найдется такая точка x = с , a c b, что

    Производные различных порядков.

    Пусть функция y = f (x ) дифференцируема на некотором отрезке [a , b ]. Значения производной f ў(x ), вообще говоря, зависят от x , т.е. производная f ў(x ) представляет собой тоже функцию от x . При дифференцировании этой функции получается так называемая вторая производная от функции f (x ), которая обозначается f ўў (x ).

    Производной n- го порядка от функции f (x ) называется производная (первого порядка) от производной n- 1го и обозначается символом y (n ) = (y (n – 1))ў.

    Дифференциалы различных порядков.

    Дифференциал функции y = f (x ), где x – независимая переменная, есть dy = f ў(x )dx , некоторая функция от x , но от x может зависеть только первый сомножитель f ў(x ), второй же сомножитель (dx ) является приращением независимой переменной x и от значения этой переменной не зависит. Так как dy есть функция от x , то можно определить дифференциал этой функции. Дифференциал от дифференциала функции называется вторым дифференциалом или дифференциалом второго порядка этой функции и обозначается d 2y :

    d (dx ) = d 2y = f ўў(x )(dx ) 2 .

    Дифференциалом n- го порядка называется первый дифференциал от дифференциала n- 1го порядка:

    d n y = d (d n –1 y ) = f (n )(x )dx (n ).

    Частная производная.

    Если функция зависит не от одного, а от нескольких аргументов x i (i изменяется от 1 до n , i = 1, 2,… n ), f (x 1, x 2,… x n ), то в дифференциальном исчислении вводится понятие частной производной, которая характеризует скорость изменения функции нескольких переменных, когда изменяется только один аргумент, например, x i . Частная производная 1-ого порядка по x i определяется как обычная производная, при этом предполагается, что все аргументы, кроме x i , сохраняют постоянные значения. Для частных производных вводятся обозначения

    Определенные таким образом частные производные 1-ого порядка (как функции тех же аргументов) могут, в свою очередь, также иметь частные производные, это частные производные второго порядка и т.д. Взятые по разным аргументам такие производные называются смешанными. Непрерывные смешанные производные одного порядка не зависят от порядка дифференцирования и равны между собой.

    Анна Чугайнова

    Определение. Пусть функция \(y = f(x) \) определена в некотором интервале, содержащем внутри себя точку \(x_0 \). Дадим аргументу приращение \(\Delta x \) такое, чтобы не выйти из этого интервала. Найдем соответствующее приращение функции \(\Delta y \) (при переходе от точки \(x_0 \) к точке \(x_0 + \Delta x \)) и составим отношение \(\frac{\Delta y}{\Delta x} \). Если существует предел этого отношения при \(\Delta x \rightarrow 0 \), то указанный предел называют производной функции \(y=f(x) \) в точке \(x_0 \) и обозначают \(f”(x_0) \).

    $$ \lim_{\Delta x \to 0} \frac{\Delta y}{\Delta x} = f”(x_0) $$

    Для обозначения производной часто используют символ y”. Отметим, что y” = f(x) – это новая функция, но, естественно, связанная с функцией y = f(x), определенная во всех точках x, в которых существует указанный выше предел. Эту функцию называют так: производная функции у = f(x) .

    Геометрический смысл производной состоит в следующем. Если к графику функции у = f(x) в точке с абсциссой х=a можно провести касательную, непараллельную оси y, то f(a) выражает угловой коэффициент касательной:
    \(k = f”(a) \)

    Поскольку \(k = tg(a) \), то верно равенство \(f”(a) = tg(a) \) .

    А теперь истолкуем определение производной с точки зрения приближенных равенств. Пусть функция \(y = f(x) \) имеет производную в конкретной точке \(x \):
    $$ \lim_{\Delta x \to 0} \frac{\Delta y}{\Delta x} = f”(x) $$
    Это означает, что около точки х выполняется приближенное равенство \(\frac{\Delta y}{\Delta x} \approx f”(x) \), т.2 \) справедливо приближенное равенство \(\Delta y \approx 2x \cdot \Delta x \). Если внимательно проанализировать определение производной, то мы обнаружим, что в нем заложен алгоритм ее нахождения.

    Сформулируем его.

    Как найти производную функции у = f(x) ?

    1. Зафиксировать значение \(x \), найти \(f(x) \)
    2. Дать аргументу \(x \) приращение \(\Delta x \), перейти в новую точку \(x+ \Delta x \), найти \(f(x+ \Delta x) \)
    3. Найти приращение функции: \(\Delta y = f(x + \Delta x) – f(x) \)
    4. Составить отношение \(\frac{\Delta y}{\Delta x} \)
    5. Вычислить $$ \lim_{\Delta x \to 0} \frac{\Delta y}{\Delta x} $$
    Этот предел и есть производная функции в точке x.

    Если функция у = f(x) имеет производную в точке х, то ее называют дифференцируемой в точке х. Процедуру нахождения производной функции у = f(x) называют дифференцированием функции у = f(x).

    Обсудим такой вопрос: как связаны между собой непрерывность и дифференцируемость функции в точке.

    Пусть функция у = f(x) дифференцируема в точке х. Тогда к графику функции в точке М(х; f(x)) можно провести касательную, причем, напомним, угловой коэффициент касательной равен f”(x). Такой график не может «разрываться» в точке М, т. е. функция обязана быть непрерывной в точке х.

    Это были рассуждения «на пальцах». Приведем более строгое рассуждение. Если функция у = f(x) дифференцируема в точке х, то выполняется приближенное равенство \(\Delta y \approx f”(x) \cdot \Delta x \). Если в этом равенстве \(\Delta x \) устремить к нулю, то и \(\Delta y \) будет стремиться к нулю, а это и есть условие непрерывности функции в точке.

    Итак, если функция дифференцируема в точке х, то она и непрерывна в этой точке .

    Обратное утверждение неверно. Например: функция у = |х| непрерывна везде, в частности в точке х = 0, но касательная к графику функции в «точке стыка» (0; 0) не существует. Если в некоторой точке к графику функции нельзя провести касательную, то в этой точке не существует производная.

    Еще один пример. Функция \(y=\sqrt{x} \) непрерывна на всей числовой прямой, в том числе в точке х = 0. И касательная к графику функции существует в любой точке, в том числе в точке х = 0. Но в этой точке касательная совпадает с осью у, т. е. перпендикулярна оси абсцисс, ее уравнение имеет вид х = 0. Углового коэффициента у такой прямой нет, значит, не существует и \(f”(0) \)

    Итак, мы познакомились с новым свойством функции – дифференцируемостью. А как по графику функции можно сделать вывод о ее дифференцируемости?

    Ответ фактически получен выше. Если в некоторой точке к графику функции можно провести касательную, не перпендикулярную оси абсцисс, то в этой точке функция дифференцируема. Если в некоторой точке касательная к графику функции не существует или она перпендикулярна оси абсцисс, то в этой точке функция не дифференцируема.

    Правила дифференцирования

    Операция нахождения производной называется дифференцированием . При выполнении этой операции часто приходится работать с частными, суммами, произведениями функций, а также с «функциями функций», то есть сложными функциями.2} $$

    Производная функции одной переменной.

    Введение.

    Настоящие методические разработки предназначены для студентов факультета промышленное и гражданское строительство. Они составлены применительно к программе курса математики по разделу «Дифференциальное исчисление функций одного переменного».

    Разработки представляют собой единое методическое руководство, включающее в себя: краткие теоретические сведения; «типовые» задачи и упражнения с подробными решениями и пояснениями к этим решениям; варианты контрольной работы.

    В конце каждого параграфа дополнительные упражнения. Такая структура разработок делает их пригодными для самостоятельного овладения разделом при самой минимальной помощи со стороны преподавателя.

    Механический и геометрический смысл

    производной.

    Понятие производной является одним из самых важных понятий математического анализа.Оно возникло еще в 17 веке. Формирование понятия производной исторически связано с двумя задачами: задачей о скорости переменного движения и задачей о касательной к кривой.

    Эти задачи, несмотря на их различное содержание, приводят к одной и той же математической операции, которую нужно провести над функцией.Эта операция получила в математике специальное название. Она называется операцией дифференцирования функции. Результат операции дифференцирования называется производной.

    Итак, производной функцииy=f(x) в точкеx0 называется предел (если он существует) отношения приращения функции к приращению аргумента
    при
    .

    Производную принято обозначать так:
    .

    Таким образом, по определению

    Для обозначения производной употребляются также символы
    .

    Механический смысл производной.

    Если s=s(t) – закон прямолинейного движения материальной точки, то
    есть скорость этой точки в момент времениt.

    Геометрический смысл производной.

    Если функция y=f(x) имеет производную в точке, то угловой коэффициент касательной к графику функции в точке
    равен
    .

    Пример.

    Найдите производную функции
    в точке=2:

    1) Дадим точке =2 приращение
    . Заметим, что.

    2) Найдем приращение функции в точке =2:

    3) Составим отношение приращения функции к приращению аргумента:

    Найдем предел отношения при
    :

    .

    Таким образом,
    .

    § 2. Производные от некоторых

    простейших функций.

    Студенту необходимо научиться вычислять производные конкретных функций: y=x,y=и вообщеy=.

    Найдем производную функции у=х.

    т.е. (x)′=1.

    Найдем производную функции

    Производная

    Пусть
    тогда

    Легко заметить закономерность в выражениях производных от степенной функции
    приn=1,2,3.

    Следовательно,

    . (1)

    Эта формула справедлива для любых действительных n.

    В частности, используя формулу (1), имеем:

    ;

    .

    Пример.

    Найдите производную функции

    .

    .

    Данная функция является частным случаем функции вида

    при
    .

    Используя формулу (1), имеем

    .

    Производные функций y=sin x и y=cos x.

    Пусть y=sinx.

    Разделим на ∆x, получим

    Переходя к пределу при ∆x→0, имеем

    Пусть y=cosx .

    Переходя к пределу при ∆x→0, получим

    ;
    . (2)

    §3. Основные правила дифференцирования.

    Рассмотрим правила дифференцирования.

    Теорема 1 . Если функцииu=u(x) иv=v(x) дифференцируемы в данной точкеx,то в этой точке дифференцируема и их сумма, причем производная суммы равна сумме производных слагаемых: (u+v)”=u”+v”.(3)

    Доказательство: рассмотрим функцию y=f(x)=u(x)+v(x).

    Приращению ∆x аргумента x соответствуют приращения ∆u=u(x+∆x)-u(x), ∆v=v(x+∆x)-v(x) функций u и v. Тогда функция y получит приращение

    ∆y=f(x+∆x)-f(x)=

    =–=∆u+∆v.

    Следовательно,

    Итак, (u+v)”=u”+v”.

    Теорема 2. Если функцииu=u(x) иv=v(x) дифференцируемы в данной точкеx, то в той же точке дифференцируемо и их произведение.При этом производная произведения находится по следующей формуле: (uv)”=u”v+uv”. (4)

    Доказательство: Пусть y=uv, где u и v – некоторые дифференцируемые функции от x. Дадим x приращение ∆x;тогда u получит приращение ∆u, v получит приращение ∆v и y получит приращение ∆y.

    Имеем y+∆y=(u+∆u)(v+∆v), или

    y+∆y=uv+u∆v+v∆u+∆u∆v.

    Следовательно, ∆y=u∆v+v∆u+∆u∆v.

    Отсюда

    Переходя к пределу при ∆x→0 и учитывая, чтоuиvне зависят от ∆x, будем иметь

    Теорема 3 . Производная частного двух функций равна дроби, знаменатель которой равен квадрату делителя, а числитель- разности между произведением производной делимого на делитель и произведением делимого на производную делителя, т.е.

    Если
    то
    (5)

    Теорема 4. Производная постоянной равна нулю, т.е. если y=C, где С=const, то y”=0.

    Теорема 5. Постоянный множитель можно выносить за знак производной, т.е. если y=Cu(x), где С=const, то y”=Cu”(x).

    Пример 1.

    Найдите производную функции

    .

    Данная функция имеет вид
    , гдеu=x,v=cosx. Применяя правило дифференцирования (4), находим

    .

    Пример 2.

    Найдите производную функции

    .

    Применим формулу (5).

    Здесь
    ;
    .

    Задачи.

    Найдите производные следующих функций:

    ;

    11)

    2)
    ; 12)
    ;

    3)
    13)

    4)
    14)

    5)
    15)

    6)
    16)

    7 )
    17)

    8)
    18)

    9)
    19)

    10)
    20)

    Операция отыскания производной называется дифференцированием.

    В результате решения задач об отыскании производных у самых простых (и не очень простых) функций по определению производной как предела отношения приращения к приращению аргумента появились таблица производных и точно определённые правила дифференцирования. Первыми на ниве нахождения производных потрудились Исаак Ньютон (1643-1727) и Готфрид Вильгельм Лейбниц (1646-1716).

    Поэтому в наше время, чтобы найти производную любой функции, не надо вычислять упомянутый выше предел отношения приращения функции к приращению аргумента, а нужно лишь воспользоваться таблицей производных и правилами дифференцирования. Для нахождения производной подходит следующий алгоритм.

    Чтобы найти производную , надо выражение под знаком штриха разобрать на составляющие простые функции и определить, какими действиями (произведение, сумма, частное) связаны эти функции. Далее производные элементарных функций находим в таблице производных, а формулы производных произведения, суммы и частного – в правилах дифференцирования. Таблица производных и правила дифференцирования даны после первых двух примеров.

    Пример 1. Найти производную функции

    Решение. Из правил дифференцирования выясняем, что производная суммы функций есть сумма производных функций, т. е.

    Из таблицы производных выясняем, что производная “икса” равна единице, а производная синуса – косинусу. Подставляем эти значения в сумму производных и находим требуемую условием задачи производную:

    Пример 2. Найти производную функции

    Решение. Дифференцируем как производную суммы, в которой второе слагаемое с постоянным множителем, его можно вынести за знак производной:

    Если пока возникают вопросы, откуда что берётся, они, как правило, проясняются после ознакомления с таблицей производных и простейшими правилами дифференцирования. К ним мы и переходим прямо сейчас.

    Таблица производных простых функций

    Правила дифференцирования

    1. Производная суммы или разности
    2. Производная произведения
    2a. Производная выражения, умноженного на постоянный множитель
    3. Производная частного
    4. Производная сложной функции

    Правило 1. Если функции

    дифференцируемы в некоторой точке , то в той же точке дифференцируемы и функции

    причём

    т.е. производная алгебраической суммы функций равна алгебраической сумме производных этих функций.

    Следствие. Если две дифференцируемые функции отличаются на постоянное слагаемое, то их производные равны , т.е.

    Правило 2. Если функции

    дифференцируемы в некоторой точке , то в то же точке дифференцируемо и их произведение

    причём

    т.е. производная произведения двух функций равна сумме произведений каждой из этих функций на производную другой.

    Следствие 1. Постоянный множитель можно выносить за знак производной :

    Следствие 2. Производная произведения нескольких дифференцируемых функций равна сумме произведений производной каждого из сомножителей на все остальные.

    Например, для трёх множителей:

    Правило 3. Если функции

    дифференцируемы в некоторой точке и , то в этой точке дифференцируемо и их частное u/v , причём

    т.е. производная частного двух функций равна дроби, числитель которой есть разность произведений знаменателя на производную числителя и числителя на производную знаменателя, а знаменатель есть квадрат прежнего числителя.

    Где что искать на других страницах

    При нахождении производной произведения и частного в реальных задачах всегда требуется применять сразу несколько правил дифференцирования, поэтому больше примеров на эти производные – в статье “Производная произведения и частного функций ” .

    Замечание. Следует не путать константу (то есть, число) как слагаемое в сумме и как постоянный множитель! В случае слагаемого её производная равна нулю, а в случае постоянного множителя она выносится за знак производных. Это типичная ошибка, которая встречается на начальном этапе изучения производных, но по мере решения уже нескольких одно- двухсоставных примеров средний студент этой ошибки уже не делает.

    А если при дифференцировании произведения или частного у вас появилось слагаемое u v , в котором u – число, например, 2 или 5, то есть константа, то производная этого числа будет равна нулю и, следовательно, всё слагаемое будет равно нулю (такой случай разобран в примере 10).

    Другая частая ошибка – механическое решение производной сложной функции как производной простой функции. Поэтому производной сложной функции посвящена отдельная статья. Но сначала будем учиться находить производные простых функций.

    По ходу не обойтись без преобразований выражений. Для этого может потребоваться открыть в новых окнах пособия Действия со степенями и корнями и Действия с дробями .

    Если Вы ищете решения производных дробей со степенями и корнями, то есть, когда функция имеет вид вроде , то следуйте на занятие “Производная суммы дробей со степенями и корнями “.

    Если же перед Вами задача вроде , то Вам на занятие “Производные простых тригонометрических функций”.

    Пошаговые примеры – как найти производную

    Пример 3. Найти производную функции

    Решение. Определяем части выражения функции: всё выражение представляет произведение, а его сомножители – суммы, во второй из которых одно из слагаемых содержит постоянный множитель. Применяем правило дифференцирования произведения: производная произведения двух функций равна сумме произведений каждой из этих функций на производную другой:

    Далее применяем правило дифференцирования суммы: производная алгебраической суммы функций равна алгебраической сумме производных этих функций. В нашем случае в каждой сумме второе слагаемое со знаком минус. В каждой сумме видим и независимую переменную, производная которой равна единице, и константу (число), производная которой равна нулю. Итак, “икс” у нас превращается в единицу, а минус 5 – в ноль. Во втором выражении “икс” умножен на 2, так что двойку умножаем на ту же единицу как производную “икса”. Получаем следующие значения производных:

    Подставляем найденные производные в сумму произведений и получаем требуемую условием задачи производную всей функции:

    Пример 4. Найти производную функции

    Решение. От нас требуется найти производную частного. Применяем формулу дифференцирования частного: производная частного двух функций равна дроби, числитель которой есть разность произведений знаменателя на производную числителя и числителя на производную знаменателя, а знаменатель есть квадрат прежнего числителя. Получаем:

    Производную сомножителей в числителе мы уже нашли в примере 2. Не забудем также, что произведение, являющееся вторым сомножителем в числителе в текущем примере берётся со знаком минус:

    Если Вы ищете решения таких задач, в которых надо найти производную функции, где сплошное нагромождение корней и степеней, как, например, , то добро пожаловать на занятие “Производная суммы дробей со степенями и корнями” .

    Если же Вам нужно узнать больше о производных синусов, косинусов, тангенсов и других тригонометрических функций, то есть, когда функция имеет вид вроде , то Вам на урок “Производные простых тригонометрических функций” .

    Пример 5. Найти производную функции

    Решение. В данной функции видим произведение, один из сомножителей которых – квадратный корень из независимой переменной, с производной которого мы ознакомились в таблице производных. По правилу дифференцирования произведения и табличному значению производной квадратного корня получаем:

    Пример 6. Найти производную функции

    Решение. В данной функции видим частное, делимое которого – квадратный корень из независимой переменной. По правилу дифференцирования частного, которое мы повторили и применили в примере 4, и табличному значению производной квадратного корня получаем:

    Чтобы избавиться от дроби в числителе, умножаем числитель и знаменатель на .

    Рекомендуем также

    Наименьшее значение производной. Производная функции

    Производная функции – одна из сложных тем в школьной программе. Не каждый выпускник ответит на вопрос, что такое производная.

    В этой статье просто и понятно рассказано о том, что такое производная и для чего она нужна . Мы не будем сейчас стремиться к математической строгости изложения. Самое главное – понять смысл.

    Запомним определение:

    Производная – это скорость изменения функции.

    На рисунке – графики трех функций. Как вы думаете, какая из них быстрее растет?

    Ответ очевиден – третья. У нее самая большая скорость изменения, то есть самая большая производная.

    Вот другой пример.

    Костя, Гриша и Матвей одновременно устроились на работу. Посмотрим, как менялся их доход в течение года:

    На графике сразу все видно, не правда ли? Доход Кости за полгода вырос больше чем в два раза. И у Гриши доход тоже вырос, но совсем чуть-чуть. А доход Матвея уменьшился до нуля. Стартовые условия одинаковые, а скорость изменения функции, то есть производная , – разная. Что касается Матвея – у его дохода производная вообще отрицательна.

    Интуитивно мы без труда оцениваем скорость изменения функции. Но как же это делаем?

    На самом деле мы смотрим, насколько круто идет вверх (или вниз) график функции. Другими словами – насколько быстро меняется у с изменением х. Очевидно, что одна и та же функция в разных точках может иметь разное значение производной – то есть может меняться быстрее или медленнее.

    Производная функции обозначается .

    Покажем, как найти с помощью графика.

    Нарисован график некоторой функции . Возьмем на нем точку с абсциссой . Проведём в этой точке касательную к графику функции. Мы хотим оценить, насколько круто вверх идет график функции. Удобная величина для этого – тангенс угла наклона касательной .

    Производная функции в точке равна тангенсу угла наклона касательной, проведённой к графику функции в этой точке.

    Обратите внимание – в качестве угла наклона касательной мы берем угол между касательной и положительным направлением оси .

    Иногда учащиеся спрашивают, что такое касательная к графику функции. Это прямая, имеющая на данном участке единственную общую точку с графиком, причем так, как показано на нашем рисунке. Похоже на касательную к окружности.

    Найдем . Мы помним, что тангенс острого угла в прямоугольном треугольнике равен отношению противолежащего катета к прилежащему. Из треугольника :

    Мы нашли производную с помощью графика, даже не зная формулу функции. Такие задачи часто встречаются в ЕГЭ по математике под номером .

    Есть и другое важное соотношение. Вспомним, что прямая задается уравнением

    Величина в этом уравнении называется угловым коэффициентом прямой . Она равна тангенсу угла наклона прямой к оси .

    .

    Мы получаем, что

    Запомним эту формулу. Она выражает геометрический смысл производной.

    Производная функции в точке равна угловому коэффициенту касательной, проведенной к графику функции в этой точке.

    Другими словами, производная равна тангенсу угла наклона касательной.

    Мы уже сказали, что у одной и той же функции в разных точках может быть разная производная. Посмотрим, как же связана производная с поведением функции.

    Нарисуем график некоторой функции . Пусть на одних участках эта функция возрастает, на других – убывает, причем с разной скоростью. И пусть у этой функции будут точки максимума и минимума.

    В точке функция возрастает. Касательная к графику, проведенная в точке , образует острый угол с положительным направлением оси . Значит, в точке производная положительна.

    В точке наша функция убывает. Касательная в этой точке образует тупой угол с положительным направлением оси . Поскольку тангенс тупого угла отрицателен, в точке производная отрицательна.

    Вот что получается:

    Если функция возрастает, ее производная положительна.

    Если убывает, ее производная отрицательна.

    А что же будет в точках максимума и минимума? Мы видим, что в точках (точка максимума) и (точка минимума) касательная горизонтальна. Следовательно, тангенс угла наклона касательной в этих точках равен нулю, и производная тоже равна нулю.

    Точка – точка максимума. В этой точке возрастание функции сменяется убыванием. Следовательно, знак производной меняется в точке с «плюса» на «минус».

    В точке – точке минимума – производная тоже равна нулю, но ее знак меняется с «минуса» на «плюс».

    Вывод: с помощью производной можно узнать о поведении функции всё, что нас интересует.

    Если производная положительна, то функция возрастает.

    Если производная отрицательная, то функция убывает.

    В точке максимума производная равна нулю и меняет знак с «плюса» на «минус».

    В точке минимума производная тоже равна нулю и меняет знак с «минуса» на «плюс».

    Запишем эти выводы в виде таблицы:

    возрастаетточка максимумаубываетточка минимумавозрастает
    +00+

    Сделаем два небольших уточнения. Одно из них понадобится вам при решении задач ЕГЭ. Другое – на первом курсе, при более серьезном изучении функций и производных.

    Возможен случай, когда производная функции в какой-либо точке равна нулю, но ни максимума, ни минимума у функции в этой точке нет. Это так называемая :

    В точке касательная к графику горизонтальна, и производная равна нулю. Однако до точки функция возрастала – и после точки продолжает возрастать. Знак производной не меняется – она как была положительной, так и осталась.

    Бывает и так, что в точке максимума или минимума производная не существует. На графике это соответствует резкому излому, когда касательную в данной точке провести невозможно.

    А как найти производную, если функция задана не графиком, а формулой? В этом случае применяется

    Этот раздел содержит задачи ЕГЭ по математике на темы, связанные с исследованием функций и их производных.

    В демонстрационных вариантах ЕГЭ 2020 года они могут встретиться под номером 14 для базового уровня и под номером 7 для профильного уровня.

    Посмотрите внимательно на эти три графика функций.
    Заметили ли вы, что эти функции в некотором смысле “родственники”?
    Например, на тех участках, где график зеленой функции расположен выше нуля, красная функция возрастает. На тех участках, где график зеленой функции ниже нуля, красная функция убывает.
    Аналогичные замечания можно сделать относительно красного и синего графиков.
    Также можно заметить, что нули зеленой функции (точки x = −1 и x = 3) совпадают с точками экстремумов красного графика: при x = −1 на красном графике мы видим локальный максимум, при х = 3 на красном графике локальный минимум.
    Нетрудно заметить, что локальные максимумы и минимумы синего графика достигаются в тех же точках, где красный график проходит через значение y = 0.
    Можно сделать еще несколько выводов об особенностях поведения этих графиков, потому что они действительно связаны между собой. Посмотрите на формулы функций, расположенные под каждым из графиков, и путем вычислений убедитесь, что каждая предыдущая является производной для последующей и, соответственно, каждая следующая является одной из превообразных предыдущей функции.

    φ 1 (x ) = φ” 2 (x ) φ 2 (x ) = Φ 1 (x )
    φ 2 (x ) = φ” 3 (x ) φ 3 (x ) = Φ 2 (x )

    Вспомним, что мы знаем о производной:

    Производная функции y = f (x ) в точке х выражает скорость изменения функции в точке x .

    Физический смысл производной заключается в том, что производная выражает скорость протекания процесса, описываемого зависимостью y = f(x).

    Геометрический смысл производной заключается в том, что её значение в рассматриваемой точке равняется угловому коэффициенту касательной, проведенной к графику дифференцируемой функции в этой точке.

    А теперь пусть красного графика на рисунке нет. Допустим, что и формулы функций нам неизвестны.

    Могу ли я спросить вас о чем то, связанном с поведением функции φ 2 (x ), если известно, что она является производной функции φ 3 (x ) и первообразной функции φ 1 (x )?
    Могу. И на многие вопросы можно дать точный ответ, ведь мы знаем, что производная является характеристикой скорости изменения функции, поэтому можем судить о некоторых особенностях поведения одной из этих функций, глядя на график другой.

    Прежде, чем отвечать на следующие вопросы, прокрутите страницу вверх так, чтобы скрылся верхний рисунок, содержащий красный график. Когда ответы будут даны, верните его обратно, чтобы проверить результат. И только после этого смотрите моё решение.

    Внимание: Для усиления обучающего эффекта ответы и решения загружаются отдельно для каждой задачи последовательным нажатием кнопок на желтом фоне. (Когда задач много, кнопки могут появиться с задержкой. Если кнопок не видно совсем, проверьте, разрешен ли в вашем браузере JavaScript. )

    1) Пользуясь графиком производной φ” 2 (x ) (в нашем случае это зеленый график), определите какое из 2-ух значений функции больше φ 2 (−3) или φ 2 (−2)?

    По графику производной видно, что на участке [−3;−2] её значения строго положительны, значит функция на этом участке только возрастает, поэтому значение функции в левом конце x = −3 меньше, чем её значение в правом конце x = −2.

    Ответ: φ 2 (−3) φ 2 (−2)

    2) Пользуясь графиком первообразной Φ 2 (x ) (в нашем случае это синий график), определите какое из 2-ух значений функции больше φ 2 (−1) или φ 2 (4)?

    По графику первообразной видно, что точка x = −1 находится на участке возрастания, следовательно значение соответсвующей производной положительно. Точка x = 4 находится на участке убывания и значение соответствующей производной отрицательно. Поскольку положительное значение больше отрицательного, делаем вывод – значение неизвестной функции, которая как раз и является производной, в точке 4 меньше, чем в точке −1.

    Ответ: φ 2 (−1) > φ 2 (4)

    Подобных вопросов по отсутствующему графику можно задать много, что обуславливает большое разноообразие задач с кратким ответом, построенных по такой же схеме. Попробуйте решить некоторые из них.

    Задачи на определение характеристик производной по графику функции.


    Рисунок 1.


    Рисунок 2.

    Задача 1

    y = f (x ), определенной на интервале (−10,5;19). Определите количество целых точек, в которых производная функции положительна.

    Производная функции положительна на тех участках, где функция возрастает. По рисунку видно, что это промежутки (−10,5;−7,6), (−1;8,2) и (15,7;19). Перечислим целые точки внутри этих интервалов: “−10″,”−9”, “−8″,”0”, “1”,”2″, “3”,”4″, “5”,”6″, “7”,”8″, “16”,”17″, “18”. Всего 15 точек.

    Ответ: 15

    Замечания.
    1. Когда в задачах о графиках функций требуют назвать “точки”, как правило, имеют в виду только значения аргумента x , которые являются абсциссами соответствующих точек, расположенных на графике. Ординаты этих точек – значения функции, они являются зависимыми и могут быть легко вычислены при необходимости.
    2. При перечислении точек мы не учитывали края интервалов, так как функция в этих точках не возрастает и не убывает, а “разворачивается”. Производная в таких точках не положительна и не отрицательна, она равна нулю, поэтому они называются стационарными точками. Кроме того, мы не рассматриваем здесь границы области определения, потому что в условии сказано, что это интервал.

    Задача 2

    На рисунке 1 изображен график функции y = f (x ), определенной на интервале (−10,5;19). Определите количество целых точек, в которых производная функции f ” (x ) отрицательна.

    Производная функции отрицательна на тех участках, где функция убывает. По рисунку видно, что это промежутки (−7,6;−1) и (8,2;15,7). Целые точки внутри этих интервалов: “−7″,”−6”, “−5″,”−4”, “−3″,”−2”, “9”,”10″, “11”,”12″, “13”,”14″, “15”. Всего 13 точек.

    Ответ: 13

    См. замечания к предыдущей задаче.

    Для решения следующих задач нужно вспомнить еще одно определение.

    Точки максимума и минимума функции объединяются общим названием – точки экстремума .

    В этих точках производная функции либо равна нулю, либо не существует (необходимое условие экстремума ).
    Однако необходимое условие – это признак, но не гарантия существования экстремума функции. Достаточным условием экстремума является смена знака производной: если производная в точке меняет знак с “+” на “−”, то это точка максимума функции; если производная в точке меняет знак с “−” на “+” , то это точка минимума функции; если в точке производная функции равна нулю, либо не существует, но знак производной при переходе через эту точку не меняется на противоположный, то указанная точка не является точкой экстремума функции. Это может быть точка перегиба, точка разрыва или точка излома графика функции.

    Задача 3

    На рисунке 1 изображен график функции y = f (x ), определенной на интервале (−10,5;19). Найдите количество точек, в которых касательная к графику функции параллельна прямой y = 6 или совпадает с ней.

    Вспомним, что уравнение прямой имеет вид y = kx + b , где k – коэффициент наклона этой прямой к оси Ox . В нашем случае k = 0, т.е. прямая y = 6 не наклонена, а параллельна оси Ox . Значит искомые касательные также должны быть параллельны оси Ox и также должны иметь коэффициент наклона 0. Таким свойством касательные обладают в точках экстремумов функций. Поэтому для ответа на вопрос нужно просто посчитать все точки экстремумов на графике. Здесь их 4 – две точки максимума и две точки минимума.

    Ответ: 4

    Задача 4

    Функции y = f (x ), определенной на интервале (−11;23). Найдите сумму точек экстремума функции на отрезке .

    На указанном отрезке мы видим 2 точки экстремума. Максимум функции достигается в точке x 1 = 4, минимум в точке x 2 = 8.
    x 1 + x 2 = 4 + 8 = 12.

    Ответ: 12

    Задача 5

    На рисунке 1 изображен график функции y = f (x ), определенной на интервале (−10,5;19). Найдите количество точек, в которых производная функции f ” (x ) равна 0.

    Производная функции равна нулю в точках экстремума, которых на графике видно 4:
    2 точки максимума и 2 точки минимума.

    Ответ: 4

    Задачи на определение характеристик функции по графику её производной.


    Рисунок 1.

    Рисунок 2.

    Задача 6

    На рисунке 2 изображен график f ” (x ) – производной функции f (x ), определенной на интервале (−11;23). В какой точке отрезка [−6;2] функция f (x ) принимает наибольшее значение.

    На указанном отрезке производная нигде не была положительной, следовательно функция не возрастала. Она убывала или проходила через стационарные точки. Таким образом, наибольшего значения функция достигала на левой границе отрезка: x = −6.

    Ответ: −6

    Замечание: По графику производной видно, что на отрезке [−6;2] она равна нулю трижды: в точках x = −6, x = −2, x = 2. Но в точке x = −2 она не меняла знака, значит в этой точке не могло быть экстремума функции. Скорее всего там была точка перегиба графика исходной функции.

    Задача 7

    На рисунке 2 изображен график f ” (x ) – производной функции f (x ), определенной на интервале (−11;23). В какой точке отрезка функция принимает наименьшее значение.

    На отрезке производная строго положительна, следовательно функция на этом участке только возрастала. Таким образом, наименьшего значения функция достигала на левой границе отрезка: x = 3.

    Ответ: 3

    Задача 8

    На рисунке 2 изображен график f ” (x ) – производной функции f (x ), определенной на интервале (−11;23). Найдите количество точек максимума функции f (x ), принадлежащих отрезку [−5;10].

    Согласно необходимому условию экстремума максимум функции может быть в точках, где её производная равна нулю. На заданном отрезке это точки: x = −2, x = 2, x = 6, x = 10. Но согласно достаточному условию он точно будет только в тех из них, где знак производной меняется с “+” на “−”. На графике производной мы видим, что из перечисленных точек такой является только точка x = 6.

    Ответ: 1

    Задача 9

    На рисунке 2 изображен график f ” (x ) – производной функции f (x ), определенной на интервале (−11;23). Найдите количество точек экстремума функции f (x ), принадлежащих отрезку .

    Экстремумы функции могут быть в тех точках, где её производная равна 0. На заданном отрезке графика производной мы видим 5 таких точек: x = 2, x = 6, x = 10, x = 14, x = 18. Но в точке x = 14 производная не поменяла знак, следовательно её надо исключить из рассмотрения. Таким образом, остаются 4 точки.

    Ответ: 4

    Задача 10

    На рисунке 1 изображен график f ” (x ) – производной функции f (x ), определенной на интервале (−10,5;19). Найдите промежутки возрастания функции f (x ). В ответе укажите длину наибольшего из них.

    Промежутки возрастания функции совпадают с промежутками положительности производной. На графике мы видим их три – (−9;−7), (4;12), (18;19). Самый длинный из них второй. Его длина l = 12 − 4 = 8.

    Ответ: 8

    Задача 11

    На рисунке 2 изображен график f ” (x ) – производной функции f (x ), определенной на интервале (−11;23). Найдите количество точек, в которых касательная к графику функции f (x ) параллельна прямой y = −2x − 11 или совпадает с ней.

    Угловой коэффициент (он же тангенс угла наклона) заданной прямой k = −2. Нас интересуют параллельные или совпадающие касательные, т.е. прямые с таким же наклоном. Исходя из геометрического смысла производной – угловой коэффициент касательной в рассматриваемой точке графика функции, пересчитываем точки, в которых производная равна −2. На рисунке 2 таких точек 9. Их удобно считать по пересечениям графика и линии координатной сетки, проходящей через значение −2 на оси Oy .

    Ответ: 9

    Как видите, по одному и тому же графику можно задать самые разнообразные вопросы о поведении функции и её производной. Также один тот же вопрос можно отнести к графикам разных функций. Будьте внимательны при решении этой задачи на экзамене, и она покажется Вам очень легкой. Другие виды задач этого задания – на геометрический смысл первообразной – будут рассмотрены в другом разделе.

    Сергей Никифоров

    Если производная функции знакопостоянна на интервале, а сама функция непрерывна на его границах, то граничные точки при­со­еди­ня­ют­ся как к про­ме­жут­кам воз­рас­та­ния, так и к про­ме­жут­кам убы­ва­ния, что полностью соответствует определению возрастающих и убывающих функций.

    Фарит Ямаев 26.10.2016 18:50

    Здравствуйте. Как же (на каком основании) можно утверждать, что в точке, где производная равна нулю, функция возрастает. Приведите доводы. Иначе, это просто чей-то каприз. По какой теореме? А также доказательство. Спасибо.

    Служба поддержки

    Значение производной в точке не имеет прямого отношения к возрастанию функции на промежутке. Рассмотрите, например, функции – все они возрастают на отрезке

    Владлен Писарев 02.11.2016 22:21

    Если функция возрастает на интервале (а;b) и определена и непрерывна в точках а и b, то она возрастает на отрезке . Т.е. точка x=2 входит в данный промежуток.

    Хотя, как правило возрастание и убывание рассматривается не на отрезке, а на интервале.

    Но в самой точке x=2, функция имеет локальный минимум. И как объяснять детям, что когда они ищут точки возрастания (убывания), то точки локального экстремума не считаем, а в промежутки возрастания (убывания) – входят.

    Учитывая, что первая часть ЕГЭ для “средней группы детского сада”, то наверное такие нюансы- перебор.

    Отдельно, большое спасибо за “Решу ЕГЭ” всем сотрудникам- отличное пособие.

    Сергей Никифоров

    Простое объяснение можно получить, если отталкиваться от определения возрастающей/убывающей функции. Напомню, что звучит оно так: функция называется возрастающей/убывающей на промежутке, если большему аргументу функции соответствует большее/меньшее значение функции. Такое определение никак не использует понятие производной, поэтому вопросов о точках, где производная обращается в ноль возникнуть не может.

    Ирина Ишмакова 20.11.2017 11:46

    Добрый день. Здесь в комментариях я вижу убеждения, что границы включать нужно. Допустим, я с этим соглашусь. Но посмотрите, пожалуйста, ваше решение к задаче 7089. Там при указании промежутков возрастания границы не включаются. И это влияет на ответ. Т.е. решения заданий 6429 и 7089 противоречат друг другу. Проясните, пожалуйста, эту ситуацию.

    Александр Иванов

    В заданиях 6429 и 7089 совершенно разные вопросы.

    В одном про промежутки возрастания, а в другом про промежутки с положительной производной.

    Противоречия нет.

    Экстремумы входят в промежутки возрастания и убывания, но точки, в которых производная равна нулю, не входят в промежутки, на которых производная положительна.

    A Z 28.01.2019 19:09

    Коллеги, есть понятие возрастания в точке

    (см. Фихтенгольц например)

    и ваше понимание возрастания в точке x=2 противочет классическому определению.

    Возрастание и убывание это процесс и хотелось бы придерживаться этого принципа.

    В любом интервале, который содержит точку x=2, функция не является возрастающей. Поэтому включение данный точки x=2 процесс особый.

    Обычно, чтобы избежать путаницы о включении концов интервалов говорят отдельно.

    Александр Иванов

    Функция y=f(x) называется возрастающей на некотором промежутке, если бо́льшему значению аргумента из этого промежутка соответствует бо́льшее значение функции.

    В точке х=2 функция дифференцируема, а на интервале (2; 6) производная положительна, значит, на промежутке }

    Наименьшая точка производной. Производная функции. Геометрический смысл производной. Вычисление значения производной. Метод двух точек

    Производная функции – одна из сложных тем в школьной программе. Не каждый выпускник ответит на вопрос, что такое производная.

    В этой статье просто и понятно рассказано о том, что такое производная и для чего она нужна . Мы не будем сейчас стремиться к математической строгости изложения. Самое главное – понять смысл.

    Запомним определение:

    Производная – это скорость изменения функции.

    На рисунке – графики трех функций. Как вы думаете, какая из них быстрее растет?

    Ответ очевиден – третья. У нее самая большая скорость изменения, то есть самая большая производная.

    Вот другой пример.

    Костя, Гриша и Матвей одновременно устроились на работу. Посмотрим, как менялся их доход в течение года:

    На графике сразу все видно, не правда ли? Доход Кости за полгода вырос больше чем в два раза. И у Гриши доход тоже вырос, но совсем чуть-чуть. А доход Матвея уменьшился до нуля. Стартовые условия одинаковые, а скорость изменения функции, то есть производная , – разная. Что касается Матвея – у его дохода производная вообще отрицательна.

    Интуитивно мы без труда оцениваем скорость изменения функции. Но как же это делаем?

    На самом деле мы смотрим, насколько круто идет вверх (или вниз) график функции. Другими словами – насколько быстро меняется у с изменением х. Очевидно, что одна и та же функция в разных точках может иметь разное значение производной – то есть может меняться быстрее или медленнее.

    Производная функции обозначается .

    Покажем, как найти с помощью графика.

    Нарисован график некоторой функции . Возьмем на нем точку с абсциссой . Проведём в этой точке касательную к графику функции. Мы хотим оценить, насколько круто вверх идет график функции. Удобная величина для этого – тангенс угла наклона касательной .

    Производная функции в точке равна тангенсу угла наклона касательной, проведённой к графику функции в этой точке.

    Обратите внимание – в качестве угла наклона касательной мы берем угол между касательной и положительным направлением оси .

    Иногда учащиеся спрашивают, что такое касательная к графику функции. Это прямая, имеющая на данном участке единственную общую точку с графиком, причем так, как показано на нашем рисунке. Похоже на касательную к окружности.

    Найдем . Мы помним, что тангенс острого угла в прямоугольном треугольнике равен отношению противолежащего катета к прилежащему. Из треугольника :

    Мы нашли производную с помощью графика, даже не зная формулу функции. Такие задачи часто встречаются в ЕГЭ по математике под номером .

    Есть и другое важное соотношение. Вспомним, что прямая задается уравнением

    Величина в этом уравнении называется угловым коэффициентом прямой . Она равна тангенсу угла наклона прямой к оси .

    .

    Мы получаем, что

    Запомним эту формулу. Она выражает геометрический смысл производной.

    Производная функции в точке равна угловому коэффициенту касательной, проведенной к графику функции в этой точке.

    Другими словами, производная равна тангенсу угла наклона касательной.

    Мы уже сказали, что у одной и той же функции в разных точках может быть разная производная. Посмотрим, как же связана производная с поведением функции.

    Нарисуем график некоторой функции . Пусть на одних участках эта функция возрастает, на других – убывает, причем с разной скоростью. И пусть у этой функции будут точки максимума и минимума.

    В точке функция возрастает. Касательная к графику, проведенная в точке , образует острый угол с положительным направлением оси . Значит, в точке производная положительна.

    В точке наша функция убывает. Касательная в этой точке образует тупой угол с положительным направлением оси . Поскольку тангенс тупого угла отрицателен, в точке производная отрицательна.

    Вот что получается:

    Если функция возрастает, ее производная положительна.

    Если убывает, ее производная отрицательна.

    А что же будет в точках максимума и минимума? Мы видим, что в точках (точка максимума) и (точка минимума) касательная горизонтальна. Следовательно, тангенс угла наклона касательной в этих точках равен нулю, и производная тоже равна нулю.

    Точка – точка максимума. В этой точке возрастание функции сменяется убыванием. Следовательно, знак производной меняется в точке с «плюса» на «минус».

    В точке – точке минимума – производная тоже равна нулю, но ее знак меняется с «минуса» на «плюс».

    Вывод: с помощью производной можно узнать о поведении функции всё, что нас интересует.

    Если производная положительна, то функция возрастает.

    Если производная отрицательная, то функция убывает.

    В точке максимума производная равна нулю и меняет знак с «плюса» на «минус».

    В точке минимума производная тоже равна нулю и меняет знак с «минуса» на «плюс».

    Запишем эти выводы в виде таблицы:

    возрастаетточка максимумаубываетточка минимумавозрастает
    +00+

    Сделаем два небольших уточнения. Одно из них понадобится вам при решении задач ЕГЭ. Другое – на первом курсе, при более серьезном изучении функций и производных.

    Возможен случай, когда производная функции в какой-либо точке равна нулю, но ни максимума, ни минимума у функции в этой точке нет. Это так называемая :

    В точке касательная к графику горизонтальна, и производная равна нулю. Однако до точки функция возрастала – и после точки продолжает возрастать. Знак производной не меняется – она как была положительной, так и осталась.

    Бывает и так, что в точке максимума или минимума производная не существует. На графике это соответствует резкому излому, когда касательную в данной точке провести невозможно.

    А как найти производную, если функция задана не графиком, а формулой? В этом случае применяется

    В задаче B9 дается график функции или производной, по которому требуется определить одну из следующих величин:

    1. Значение производной в некоторой точке x 0 ,
    2. Точки максимума или минимума (точки экстремума),
    3. Интервалы возрастания и убывания функции (интервалы монотонности).

    Функции и производные, представленные в этой задаче, всегда непрерывны, что значительно упрощает решение. Не смотря на то, что задача относится к разделу математического анализа, она вполне по силам даже самым слабым ученикам, поскольку никаких глубоких теоретических познаний здесь не требуется.

    Для нахождения значения производной, точек экстремума и интервалов монотонности существуют простые и универсальные алгоритмы — все они будут рассмотрены ниже.

    Внимательно читайте условие задачи B9, чтобы не допускать глупых ошибок: иногда попадаются довольно объемные тексты, но важных условий, которые влияют на ход решения, там немного.

    Вычисление значения производной. Метод двух точек

    Если в задаче дан график функции f(x), касательная к этому графику в некоторой точке x 0 , и требуется найти значение производной в этой точке, применяется следующий алгоритм:

    1. Найти на графике касательной две «адекватные» точки: их координаты должны быть целочисленными. Обозначим эти точки A (x 1 ; y 1) и B (x 2 ; y 2). Правильно выписывайте координаты — это ключевой момент решения, и любая ошибка здесь приводит к неправильному ответу.
    2. Зная координаты, легко вычислить приращение аргумента Δx = x 2 − x 1 и приращение функции Δy = y 2 − y 1 .
    3. Наконец, находим значение производной D = Δy/Δx. Иными словами, надо разделить приращение функции на приращение аргумента — и это будет ответ.

    Еще раз отметим: точки A и B надо искать именно на касательной, а не на графике функции f(x), как это часто случается. Касательная обязательно будет содержать хотя бы две таких точки — иначе задача составлена некорректно.

    Рассмотрим точки A (−3; 2) и B (−1; 6) и найдем приращения:
    Δx = x 2 − x 1 = −1 − (−3) = 2; Δy = y 2 − y 1 = 6 − 2 = 4.

    Найдем значение производной: D = Δy/Δx = 4/2 = 2.

    Задача. На рисунке изображен график функции y = f(x) и касательная к нему в точке с абсциссой x 0 . Найдите значение производной функции f(x) в точке x 0 .

    Рассмотрим точки A (0; 3) и B (3; 0), найдем приращения:
    Δx = x 2 − x 1 = 3 − 0 = 3; Δy = y 2 − y 1 = 0 − 3 = −3.

    Теперь находим значение производной: D = Δy/Δx = −3/3 = −1.

    Задача. На рисунке изображен график функции y = f(x) и касательная к нему в точке с абсциссой x 0 . Найдите значение производной функции f(x) в точке x 0 .

    Рассмотрим точки A (0; 2) и B (5; 2) и найдем приращения:
    Δx = x 2 − x 1 = 5 − 0 = 5; Δy = y 2 − y 1 = 2 − 2 = 0.

    Осталось найти значение производной: D = Δy/Δx = 0/5 = 0.

    Из последнего примера можно сформулировать правило: если касательная параллельна оси OX, производная функции в точке касания равна нулю. В этом случае даже не надо ничего считать — достаточно взглянуть на график.

    Вычисление точек максимума и минимума

    Иногда вместо графика функции в задаче B9 дается график производной и требуется найти точку максимума или минимума функции. При таком раскладе метод двух точек бесполезен, но существует другой, еще более простой алгоритм. Для начала определимся с терминологией:

    1. Точка x 0 называется точкой максимума функции f(x), если в некоторой окрестности этой точки выполняется неравенство: f(x 0) ≥ f(x).
    2. Точка x 0 называется точкой минимума функции f(x), если в некоторой окрестности этой точки выполняется неравенство: f(x 0) ≤ f(x).

    Для того чтобы найти точки максимума и минимума по графику производной, достаточно выполнить следующие шаги:

    1. Перечертить график производной, убрав всю лишнюю информацию. Как показывает практика, лишние данные только мешают решению. Поэтому отмечаем на координатной оси нули производной — и все.
    2. Выяснить знаки производной на промежутках между нулями. Если для некоторой точки x 0 известно, что f’(x 0) ≠ 0, то возможны лишь два варианта: f’(x 0) ≥ 0 или f’(x 0) ≤ 0. Знак производной легко определить по исходному чертежу: если график производной лежит выше оси OX, значит f’(x) ≥ 0. И наоборот, если график производной проходит под осью OX, то f’(x) ≤ 0.
    3. Снова проверяем нули и знаки производной. Там, где знак меняется с минуса на плюс, находится точка минимума. И наоборот, если знак производной меняется с плюса на минус, это точка максимума. Отсчет всегда ведется слева направо.

    Эта схема работает только для непрерывных функций — других в задаче B9 не встречается.

    Задача. На рисунке изображен график производной функции f(x), определенной на отрезке [−5; 5]. Найдите точку минимума функции f(x) на этом отрезке.

    Избавимся от лишней информации — оставим только границы [−5; 5] и нули производной x = −3 и x = 2,5. Также отметим знаки:

    Очевидно, в точке x = −3 знак производной меняется с минуса на плюс. Это и есть точка минимума.

    Задача. На рисунке изображен график производной функции f(x), определенной на отрезке [−3; 7]. Найдите точку максимума функции f(x) на этом отрезке.

    Перечертим график, оставив на координатной оси только границы [−3; 7] и нули производной x = −1,7 и x = 5. Отметим на полученном графике знаки производной. Имеем:

    Очевидно, в точке x = 5 знак производной меняется с плюса на минус — это точка максимума.

    Задача. На рисунке изображен график производной функции f(x), определенной на отрезке [−6; 4]. Найдите количество точек максимума функции f(x), принадлежащих отрезку [−4; 3].

    Из условия задачи следует, что достаточно рассмотреть только часть графика, ограниченную отрезком [−4; 3]. Поэтому строим новый график, на котором отмечаем только границы [−4; 3] и нули производной внутри него. А именно, точки x = −3,5 и x = 2. Получаем:

    На этом графике есть лишь одна точка максимума x = 2. Именно в ней знак производной меняется с плюса на минус.

    Небольшое замечание по поводу точек с нецелочисленными координатами. Например, в последней задаче была рассмотрена точка x = −3,5, но с тем же успехом можно взять x = −3,4. Если задача составлена корректно, такие изменения не должны влиять на ответ, поскольку точки «без определенного места жительства» не принимают непосредственного участия в решении задачи. Разумеется, с целочисленными точками такой фокус не пройдет.

    Нахождение интервалов возрастания и убывания функции

    В такой задаче, подобно точкам максимума и минимума, предлагается по графику производной отыскать области, в которых сама функция возрастает или убывает. Для начала определим, что такое возрастание и убывание:

    1. Функция f(x) называется возрастающей на отрезке если для любых двух точек x 1 и x 2 из этого отрезка верно утверждение: x 1 ≤ x 2 ⇒ f(x 1) ≤ f(x 2). Другими словами, чем больше значение аргумента, тем больше значение функции.
    2. Функция f(x) называется убывающей на отрезке если для любых двух точек x 1 и x 2 из этого отрезка верно утверждение: x 1 ≤ x 2 ⇒ f(x 1) ≥ f(x 2). Т.е. большему значению аргумента соответствует меньшее значение функции.

    Сформулируем достаточные условия возрастания и убывания:

    1. Для того чтобы непрерывная функция f(x) возрастала на отрезке , достаточно, чтобы ее производная внутри отрезка была положительна, т.е. f’(x) ≥ 0.
    2. Для того чтобы непрерывная функция f(x) убывала на отрезке , достаточно, чтобы ее производная внутри отрезка была отрицательна, т.е. f’(x) ≤ 0.

    Примем эти утверждения без доказательств. Таким образом, получаем схему для нахождения интервалов возрастания и убывания, которая во многом похожа на алгоритм вычисления точек экстремума:

    1. Убрать всю лишнюю информацию. На исходном графике производной нас интересуют в первую очередь нули функции, поэтому оставим только их.
    2. Отметить знаки производной на интервалах между нулями. Там, где f’(x) ≥ 0, функция возрастает, а где f’(x) ≤ 0 — убывает. Если в задаче установлены ограничения на переменную x, дополнительно отмечаем их на новом графике.
    3. Теперь, когда нам известно поведение функции и ограничения, остается вычислить требуемую в задаче величину.

    Задача. На рисунке изображен график производной функции f(x), определенной на отрезке [−3; 7,5]. Найдите промежутки убывания функции f(x). В ответе укажите сумму целых чисел, входящих в эти промежутки.

    Как обычно, перечертим график и отметим границы [−3; 7,5], а также нули производной x = −1,5 и x = 5,3. Затем отметим знаки производной. Имеем:

    Поскольку на интервале (− 1,5) производная отрицательна, это и есть интервал убывания функции. Осталось просуммировать все целые числа, которые находятся внутри этого интервала:
    −1 + 0 + 1 + 2 + 3 + 4 + 5 = 14.

    Задача. На рисунке изображен график производной функции f(x), определенной на отрезке [−10; 4]. Найдите промежутки возрастания функции f(x). В ответе укажите длину наибольшего из них.

    Избавимся от лишней информации. Оставим только границы [−10; 4] и нули производной, которых в этот раз оказалось четыре: x = −8, x = −6, x = −3 и x = 2. Отметим знаки производной и получим следующую картинку:

    Нас интересуют промежутки возрастания функции, т.е. такие, где f’(x) ≥ 0. На графике таких промежутков два: (−8; −6) и (−3; 2). Вычислим их длины:
    l 1 = − 6 − (−8) = 2;
    l 2 = 2 − (−3) = 5.

    Поскольку требуется найти длину наибольшего из интервалов, в ответ записываем значение l 2 = 5.

    Сергей Никифоров

    Если производная функции знакопостоянна на интервале, а сама функция непрерывна на его границах, то граничные точки при­со­еди­ня­ют­ся как к про­ме­жут­кам воз­рас­та­ния, так и к про­ме­жут­кам убы­ва­ния, что полностью соответствует определению возрастающих и убывающих функций.

    Фарит Ямаев 26.10.2016 18:50

    Здравствуйте. Как же (на каком основании) можно утверждать, что в точке, где производная равна нулю, функция возрастает. Приведите доводы. Иначе, это просто чей-то каприз. По какой теореме? А также доказательство. Спасибо.

    Служба поддержки

    Значение производной в точке не имеет прямого отношения к возрастанию функции на промежутке. Рассмотрите, например, функции – все они возрастают на отрезке

    Владлен Писарев 02.11.2016 22:21

    Если функция возрастает на интервале (а;b) и определена и непрерывна в точках а и b, то она возрастает на отрезке . Т.е. точка x=2 входит в данный промежуток.

    Хотя, как правило возрастание и убывание рассматривается не на отрезке, а на интервале.

    Но в самой точке x=2, функция имеет локальный минимум. И как объяснять детям, что когда они ищут точки возрастания (убывания), то точки локального экстремума не считаем, а в промежутки возрастания (убывания) – входят.

    Учитывая, что первая часть ЕГЭ для “средней группы детского сада”, то наверное такие нюансы- перебор.

    Отдельно, большое спасибо за “Решу ЕГЭ” всем сотрудникам- отличное пособие.

    Сергей Никифоров

    Простое объяснение можно получить, если отталкиваться от определения возрастающей/убывающей функции. Напомню, что звучит оно так: функция называется возрастающей/убывающей на промежутке, если большему аргументу функции соответствует большее/меньшее значение функции. Такое определение никак не использует понятие производной, поэтому вопросов о точках, где производная обращается в ноль возникнуть не может.

    Ирина Ишмакова 20.11.2017 11:46

    Добрый день. Здесь в комментариях я вижу убеждения, что границы включать нужно. Допустим, я с этим соглашусь. Но посмотрите, пожалуйста, ваше решение к задаче 7089. Там при указании промежутков возрастания границы не включаются. И это влияет на ответ. Т.е. решения заданий 6429 и 7089 противоречат друг другу. Проясните, пожалуйста, эту ситуацию.

    Александр Иванов

    В заданиях 6429 и 7089 совершенно разные вопросы.

    В одном про промежутки возрастания, а в другом про промежутки с положительной производной.

    Противоречия нет.

    Экстремумы входят в промежутки возрастания и убывания, но точки, в которых производная равна нулю, не входят в промежутки, на которых производная положительна.

    A Z 28.01.2019 19:09

    Коллеги, есть понятие возрастания в точке

    (см. Фихтенгольц например)

    и ваше понимание возрастания в точке x=2 противочет классическому определению.

    Возрастание и убывание это процесс и хотелось бы придерживаться этого принципа.

    В любом интервале, который содержит точку x=2, функция не является возрастающей. Поэтому включение данный точки x=2 процесс особый.

    Обычно, чтобы избежать путаницы о включении концов интервалов говорят отдельно.

    Александр Иванов

    Функция y=f(x) называется возрастающей на некотором промежутке, если бо́льшему значению аргумента из этого промежутка соответствует бо́льшее значение функции.

    В точке х=2 функция дифференцируема, а на интервале (2; 6) производная положительна, значит, на промежутке её значения строго положительны, значит функция на этом участке только возрастает, поэтому значение функции в левом конце x = −3 меньше, чем её значение в правом конце x = −2.

    Ответ: φ 2 (−3) φ 2 (−2)

    2) Пользуясь графиком первообразной Φ 2 (x ) (в нашем случае это синий график), определите какое из 2-ух значений функции больше φ 2 (−1) или φ 2 (4)?

    По графику первообразной видно, что точка x = −1 находится на участке возрастания, следовательно значение соответсвующей производной положительно. Точка x = 4 находится на участке убывания и значение соответствующей производной отрицательно. Поскольку положительное значение больше отрицательного, делаем вывод – значение неизвестной функции, которая как раз и является производной, в точке 4 меньше, чем в точке −1.

    Ответ: φ 2 (−1) > φ 2 (4)

    Подобных вопросов по отсутствующему графику можно задать много, что обуславливает большое разноообразие задач с кратким ответом, построенных по такой же схеме. Попробуйте решить некоторые из них.

    Задачи на определение характеристик производной по графику функции.


    Рисунок 1.


    Рисунок 2.

    Задача 1

    y = f (x ), определенной на интервале (−10,5;19). Определите количество целых точек, в которых производная функции положительна.

    Производная функции положительна на тех участках, где функция возрастает. По рисунку видно, что это промежутки (−10,5;−7,6), (−1;8,2) и (15,7;19). Перечислим целые точки внутри этих интервалов: “−10″,”−9”, “−8″,”0”, “1”,”2″, “3”,”4″, “5”,”6″, “7”,”8″, “16”,”17″, “18”. Всего 15 точек.

    Ответ: 15

    Замечания.
    1. Когда в задачах о графиках функций требуют назвать “точки”, как правило, имеют в виду только значения аргумента x , которые являются абсциссами соответствующих точек, расположенных на графике. Ординаты этих точек – значения функции, они являются зависимыми и могут быть легко вычислены при необходимости.
    2. При перечислении точек мы не учитывали края интервалов, так как функция в этих точках не возрастает и не убывает, а “разворачивается”. Производная в таких точках не положительна и не отрицательна, она равна нулю, поэтому они называются стационарными точками. Кроме того, мы не рассматриваем здесь границы области определения, потому что в условии сказано, что это интервал.

    Задача 2

    На рисунке 1 изображен график функции y = f (x ), определенной на интервале (−10,5;19). Определите количество целых точек, в которых производная функции f ” (x ) отрицательна.

    Производная функции отрицательна на тех участках, где функция убывает. По рисунку видно, что это промежутки (−7,6;−1) и (8,2;15,7). Целые точки внутри этих интервалов: “−7″,”−6”, “−5″,”−4”, “−3″,”−2”, “9”,”10″, “11”,”12″, “13”,”14″, “15”. Всего 13 точек.

    Ответ: 13

    См. замечания к предыдущей задаче.

    Для решения следующих задач нужно вспомнить еще одно определение.

    Точки максимума и минимума функции объединяются общим названием – точки экстремума .

    В этих точках производная функции либо равна нулю, либо не существует (необходимое условие экстремума ).
    Однако необходимое условие – это признак, но не гарантия существования экстремума функции. Достаточным условием экстремума является смена знака производной: если производная в точке меняет знак с “+” на “−”, то это точка максимума функции; если производная в точке меняет знак с “−” на “+” , то это точка минимума функции; если в точке производная функции равна нулю, либо не существует, но знак производной при переходе через эту точку не меняется на противоположный, то указанная точка не является точкой экстремума функции. Это может быть точка перегиба, точка разрыва или точка излома графика функции.

    Задача 3

    На рисунке 1 изображен график функции y = f (x ), определенной на интервале (−10,5;19). Найдите количество точек, в которых касательная к графику функции параллельна прямой y = 6 или совпадает с ней.

    Вспомним, что уравнение прямой имеет вид y = kx + b , где k – коэффициент наклона этой прямой к оси Ox . В нашем случае k = 0, т.е. прямая y = 6 не наклонена, а параллельна оси Ox . Значит искомые касательные также должны быть параллельны оси Ox и также должны иметь коэффициент наклона 0. Таким свойством касательные обладают в точках экстремумов функций. Поэтому для ответа на вопрос нужно просто посчитать все точки экстремумов на графике. Здесь их 4 – две точки максимума и две точки минимума.

    Ответ: 4

    Задача 4

    Функции y = f (x ), определенной на интервале (−11;23). Найдите сумму точек экстремума функции на отрезке .

    На указанном отрезке мы видим 2 точки экстремума. Максимум функции достигается в точке x 1 = 4, минимум в точке x 2 = 8.
    x 1 + x 2 = 4 + 8 = 12.

    Ответ: 12

    Задача 5

    На рисунке 1 изображен график функции y = f (x ), определенной на интервале (−10,5;19). Найдите количество точек, в которых производная функции f ” (x ) равна 0.

    Производная функции равна нулю в точках экстремума, которых на графике видно 4:
    2 точки максимума и 2 точки минимума.

    Ответ: 4

    Задачи на определение характеристик функции по графику её производной.


    Рисунок 1.

    Рисунок 2.

    Задача 6

    На рисунке 2 изображен график f ” (x ) – производной функции f (x ), определенной на интервале (−11;23). В какой точке отрезка [−6;2] функция f (x ) принимает наибольшее значение.

    На указанном отрезке производная нигде не была положительной, следовательно функция не возрастала. Она убывала или проходила через стационарные точки. Таким образом, наибольшего значения функция достигала на левой границе отрезка: x = −6.

    Ответ: −6

    Замечание: По графику производной видно, что на отрезке [−6;2] она равна нулю трижды: в точках x = −6, x = −2, x = 2. Но в точке x = −2 она не меняла знака, значит в этой точке не могло быть экстремума функции. Скорее всего там была точка перегиба графика исходной функции.

    Задача 7

    На рисунке 2 изображен график f ” (x ) – производной функции f (x ), определенной на интервале (−11;23). В какой точке отрезка функция принимает наименьшее значение.

    На отрезке производная строго положительна, следовательно функция на этом участке только возрастала. Таким образом, наименьшего значения функция достигала на левой границе отрезка: x = 3.

    Ответ: 3

    Задача 8

    На рисунке 2 изображен график f ” (x ) – производной функции f (x ), определенной на интервале (−11;23). Найдите количество точек максимума функции f (x ), принадлежащих отрезку [−5;10].

    Согласно необходимому условию экстремума максимум функции может быть в точках, где её производная равна нулю. На заданном отрезке это точки: x = −2, x = 2, x = 6, x = 10. Но согласно достаточному условию он точно будет только в тех из них, где знак производной меняется с “+” на “−”. На графике производной мы видим, что из перечисленных точек такой является только точка x = 6.

    Ответ: 1

    Задача 9

    На рисунке 2 изображен график f ” (x ) – производной функции f (x ), определенной на интервале (−11;23). Найдите количество точек экстремума функции f (x ), принадлежащих отрезку .

    Экстремумы функции могут быть в тех точках, где её производная равна 0. На заданном отрезке графика производной мы видим 5 таких точек: x = 2, x = 6, x = 10, x = 14, x = 18. Но в точке x = 14 производная не поменяла знак, следовательно её надо исключить из рассмотрения. Таким образом, остаются 4 точки.

    Ответ: 4

    Задача 10

    На рисунке 1 изображен график f ” (x ) – производной функции f (x ), определенной на интервале (−10,5;19). Найдите промежутки возрастания функции f (x ). В ответе укажите длину наибольшего из них.

    Промежутки возрастания функции совпадают с промежутками положительности производной. На графике мы видим их три – (−9;−7), (4;12), (18;19). Самый длинный из них второй. Его длина l = 12 − 4 = 8.

    Ответ: 8

    Задача 11

    На рисунке 2 изображен график f ” (x ) – производной функции f (x ), определенной на интервале (−11;23). Найдите количество точек, в которых касательная к графику функции f (x ) параллельна прямой y = −2x − 11 или совпадает с ней.

    Угловой коэффициент (он же тангенс угла наклона) заданной прямой k = −2. Нас интересуют параллельные или совпадающие касательные, т.е. прямые с таким же наклоном. Исходя из геометрического смысла производной – угловой коэффициент касательной в рассматриваемой точке графика функции, пересчитываем точки, в которых производная равна −2. На рисунке 2 таких точек 9. Их удобно считать по пересечениям графика и линии координатной сетки, проходящей через значение −2 на оси Oy .

    Ответ: 9

    Как видите, по одному и тому же графику можно задать самые разнообразные вопросы о поведении функции и её производной. Также один тот же вопрос можно отнести к графикам разных функций. Будьте внимательны при решении этой задачи на экзамене, и она покажется Вам очень легкой. Другие виды задач этого задания – на геометрический смысл первообразной – будут рассмотрены в другом разделе.

    Функции и признаки их производных

    Его производная

    Но сначала определение. Производная функции – это наклон функции или скорость ее изменения в любой заданной точке. У прямой есть производная, постоянная на всем протяжении. Но нелинейные функции, такие как квадратичные и рациональные функции, имеют кривые с изменяющимся наклоном. Взяв производную от вашей функции, вы получите функцию для ее наклона. Затем вы можете вставить любое значение в эту производную, и вы найдете наклон в этой точке.

    Например, производная вашей функции следующая. Вы нашли производную, следуя всем правилам, которые вы уже изучили для поиска производных.

    Чтобы найти наклон, когда значение функции x равно 2, вы вставляете 2 для x в производную функцию. Затем вы оцениваете это. Вот что вы найдете.

    Это говорит о том, что, когда функция находится в точке, где x = 2 , наклон равен 1/5.

    Знак производной

    Теперь обратите внимание на знак наклона в этой точке. Это положительно. Это говорит вам о том, что ваша функция в этот момент повышается. 1/5 – это не очень большой наклон, поэтому ваша функция не будет расти очень быстро в этой точке. Вы можете увидеть это, если построите график функции. В точке x = 2 вы можете видеть, что ваша функция только начинает расти. Он не растет очень быстро, но увеличивается.

    Отрицательный наклон или отрицательная производная, с другой стороны, означает, что функция убывает.Если вы подключили 0 для x , вы обнаружите, что ваша производная отрицательна, что означает, что ваша функция уменьшается в этой точке. Ваш график, как видите, об этом свидетельствует.

    Пример

    Попробуем задачу.

    В какой области увеличивается эта функция?

    Во-первых, вам нужно взять производную, прежде чем даже пытаться ответить на вопрос.

    После получения производной вы можете теперь попытаться выяснить, в какой области увеличивается ваша функция.Вы помните, что положительная производная означает, что функция увеличивается, а отрицательная производная означает, что функция убывает. Глядя на свой вывод, вы видите, что оно просто равно 9, положительному числу. Это число не зависит от какого-либо значения x , поэтому это говорит вам, что ваша функция имеет постоянный наклон повсюду. Поскольку производная представляет собой положительное число, ваша функция увеличивается во всей области от отрицательной бесконечности до положительной бесконечности.

    Итоги урока

    Давайте рассмотрим.

    Производная функции – это наклон функции или скорость ее изменения в любой заданной точке. Чтобы найти производную функции, вы будете следовать правилам поиска производных.

    После того, как вы нашли производную, вы можете быстро определить, увеличивается или уменьшается ваша функция в данной точке. Если ваша производная отрицательна, ваша функция уменьшается. Если ваша производная положительна, ваша функция увеличивается.

    4.5 Производные и форма графа – Исчисление Том 1

    Цели обучения

    • 4.5.1 Объясните, как знак первой производной влияет на форму графика функции.
    • 4.5.2 Сформулируйте первый тест производной для критических точек.
    • 4.5.3 Используйте точки вогнутости и перегиба, чтобы объяснить, как знак второй производной влияет на форму графика функции.
    • 4.5.4 Объясните тест на вогнутость функции на открытом интервале.
    • 4.5.5 Объясните связь между функцией и ее первой и второй производными.
    • 4.5.6 Сформулируйте тест второй производной для локальных экстремумов.

    Ранее в этой главе мы заявляли, что если функция ff имеет локальный экстремум в точке c, c, то cc должна быть критической точкой f.f. Однако не гарантируется, что функция имеет локальный экстремум в критической точке. Например, f (x) = x3f (x) = x3 имеет критическую точку при x = 0x = 0, поскольку f ′ (x) = 3x2f ′ (x) = 3×2 равно нулю при x = 0, x = 0, но ff не имеет локального экстремума при x = 0.х = 0. Используя результаты из предыдущего раздела, теперь мы можем определить, действительно ли критическая точка функции соответствует локальному экстремальному значению. В этом разделе мы также увидим, как вторая производная предоставляет информацию о форме графика, описывая, изгибается ли график функции вверх или вниз.

    Первый производный тест

    Следствие 33 теоремы о среднем значении показало, что если производная функции положительна на интервале II, то функция возрастает на интервале I.I. С другой стороны, если производная функции отрицательна на интервале I, I, тогда функция убывает на интервале II, как показано на следующем рисунке.

    Фигура 4,30 Обе функции растут на интервале (a, b). (A, b). В каждой точке x, x производная f ′ (x)> 0. f ′ (x)> 0. Обе функции убывают на интервале (a, b). (A, b). В каждой точке x, x производная f ′ (x) <0. f ′ (x) <0.

    Непрерывная функция ff имеет локальный максимум в точке cc тогда и только тогда, когда ff переключается с увеличения на уменьшение в точке c.c. Точно так же ff имеет локальный минимум в точке cc тогда и только тогда, когда ff переключается с уменьшения на увеличение в точке c.c. Если ff – непрерывная функция на интервале II, содержащем cc и дифференцируемая по I, I, за исключением, возможно, точек c, c, единственный способ ff может переключиться с увеличения на уменьшение (или наоборот) в точке cc, если f′f ′ меняет знак при увеличении xx на cc Если ff дифференцируема в c, c, то это единственный способ, которым f′.f ′. может менять знак при увеличении xx на cc is, если f ′ (c) = 0. f ′ (c) = 0. Следовательно, для функции ff, которая непрерывна на интервале II, содержащем cc и дифференцируема по I, I, за исключением, возможно, точек c, c, единственный способ ff может переключиться с увеличения на уменьшение (или наоборот) – это если f ′ (c ) = 0f ′ (c) = 0 или f ′ (c) f ′ (c) не определено.Следовательно, чтобы найти локальные экстремумы для функции f, f, мы ищем точки cc в области определения ff такие, что f ′ (c) = 0f ′ (c) = 0 или f ′ (c) f ′ (c) равно неопределенный. Напомним, что такие точки называются критическими точками ф.ф.

    Обратите внимание, что ff не обязательно должен иметь локальные экстремумы в критической точке. Критические точки являются кандидатами только в локальные экстремумы. На рис. 4.31 мы показываем, что если непрерывная функция ff имеет локальный экстремум, он должен возникать в критической точке, но функция может не иметь локального экстремума в критической точке.Мы показываем, что если ff имеет локальный экстремум в критической точке, то знак f′f ′ меняется по мере увеличения xx через эту точку.

    Фигура 4,31 Функция ff имеет четыре критических точки: a, b, c, andd.a, b, c, andd. Функция ff имеет локальные максимумы в точках aa и d, d и локальный минимум в точках b.b. Функция ff не имеет локального экстремума в c.c. Знак f′f ′ меняется на всех локальных экстремумах.

    Используя рисунок 4.31, мы суммируем основные результаты, касающиеся локальных экстремумов.

    • Если непрерывная функция ff имеет локальный экстремум, он должен возникать в критической точке c.c.
    • Функция имеет локальный экстремум в критической точке cc тогда и только тогда, когда производная f′f ′ меняет знак при увеличении xx через c.c.
    • Следовательно, чтобы проверить, имеет ли функция локальный экстремум в критической точке c, c, мы должны определить знак f ′ (x) f ′ (x) слева и справа от c.c.

    Этот результат известен как тест первой производной.

    Теорема 4.9

    Первый производный тест

    Предположим, что ff – непрерывная функция на интервале II, содержащем критическую точку c.c. Если ff дифференцируема над I, I, за исключением, возможно, точки c, c, то f (c) f (c) удовлетворяет одному из следующих описаний:

    1. Если f′f ′ меняет знак с положительного, когда x c, x> c, то f (c) f (c) является локальным максимумом f.f.
    2. Если f′f ′ меняет знак с отрицательного, когда x c, x> c, то f (c) f (c) является локальным минимумом f.f.
    3. Если f′f ′ имеет один и тот же знак для x c, x> c, то f (c) f (c) не является ни локальным максимумом, ни локальным минимумом f.f.

    Мы можем резюмировать тест первой производной как стратегию поиска локальных экстремумов.

    Стратегия решения проблем

    Стратегия решения проблем: использование первого производного теста

    Рассмотрим функцию ff, непрерывную на интервале I.I.

    1. Найдите все критические точки ff и разделите интервал II на меньшие интервалы, используя критические точки в качестве конечных точек.
    2. Проанализируйте знак f′f ′ в каждом из подынтервалов.Если f′f ′ непрерывен на данном подынтервале (что обычно бывает), то знак f′f ′ на этом подынтервале не меняется и, следовательно, может быть определен путем выбора произвольной контрольной точки xx в этом подынтервале. и оценивая знак f′f ′ в этой контрольной точке. Используйте знаковый анализ, чтобы определить, увеличивается или уменьшается ff в течение этого интервала.
    3. Используйте First Derivative Test и результаты шага 22, чтобы определить, имеет ли ff локальный максимум, локальный минимум или ни один в каждой из критических точек.

    Теперь давайте посмотрим, как использовать эту стратегию для поиска всех локальных экстремумов для определенных функций.

    Пример 4,17

    Использование теста первой производной для поиска локальных экстремумов

    Используйте тест первой производной, чтобы найти расположение всех локальных экстремумов для f (x) = x3−3×2−9x − 1.f (x) = x3−3×2−9x − 1. Используйте графическую утилиту, чтобы подтвердить свои результаты.

    Решение

    Шаг 1. Производная равна f ′ (x) = 3×2−6x − 9.f ′ (x) = 3×2−6x − 9. Чтобы найти критические точки, нам нужно найти, где f ′ (x) = 0. f ′ (x) = 0. Разлагая многочлен на множители, заключаем, что критические точки должны удовлетворять

    3 (x2−2x − 3) = 3 (x − 3) (x + 1) = 0,3 (x2−2x − 3) = 3 (x − 3) (x + 1) = 0.

    Следовательно, критическими точками являются x = 3, −1.x = 3, −1. Теперь разделите интервал (−∞, ∞) (- ∞, ∞) на меньшие интервалы (−∞, −1), (- 1,3) и (3, ∞). (- ∞, −1), ( −1,3) и (3, ∞).

    Шаг 2. Поскольку f′f ′ – непрерывная функция, для определения знака f ′ (x) f ′ (x) на каждом подынтервале достаточно выбрать точку на каждом из интервалов (−∞, −1 ), (- 1,3) и (3, ∞) (- ∞, −1), (- 1,3) и (3, ∞) и определяют знак f′f ′ в каждой из этих точек.Например, давайте выберем x = −2, x = 0 и x = 4x = −2, x = 0 и x = 4 в качестве контрольных точек.

    Интервал Контрольная точка Знак f ′ (x) = 3 (x − 3) (x + 1) f ′ (x) = 3 (x − 3) (x + 1) в контрольной точке Заключение
    (−∞, −1) (- ∞, −1) х = -2 х = -2 (+) (-) (-) = + (+) (-) (-) = + ff увеличивается.
    (−1,3) (- 1,3) х = 0х = 0 (+) (-) (+) = – (+) (-) (+) = – ff уменьшается.
    (3, ∞) (3, ∞) х = 4х = 4 (+) (+) (+) = + (+) (+) (+) = + ff увеличивается.

    Шаг 3. Поскольку f′f ′ меняет знак с положительного на отрицательный, когда xx увеличивается до –1, f – 1, f имеет локальный максимум при x = −1.x = −1. Поскольку f′f ′ меняет знак с отрицательного на положительный при увеличении xx до 3, f3, f имеет локальный минимум при x = 3.x = 3. Эти аналитические результаты согласуются со следующим графиком.

    Фигура 4,32 Функция ff имеет максимум при x = −1x = −1 и минимум при x = 3x = 3.

    Контрольно-пропускной пункт 4.16

    Используйте тест первой производной, чтобы найти все локальные экстремумы для f (x) = – x3 + 32×2 + 18x.f (x) = – x3 + 32×2 + 18x.

    Пример 4,18

    Использование теста первой производной

    Используйте тест первой производной, чтобы найти расположение всех локальных экстремумов для f (x) = 5×1 / 3 − x5 / 3.f (x) = 5×1 / 3 − x5 / 3. Используйте графическую утилиту, чтобы подтвердить свои результаты.

    Решение

    Шаг 1. Производная:

    f ′ (x) = 53x − 2 / 3−53×2 / 3 = 53×2 / 3−5×2 / 33 = 5−5×4 / 33×2 / 3 = 5 (1 − x4 / 3) 3×2 / 3.f ′ (x) = 53x − 2 / 3−53×2 / 3 = 53×2 / 3−5×2 / 33 = 5−5×4 / 33×2 / 3 = 5 (1 − x4 / 3) 3×2 / 3.

    Производная f ′ (x) = 0f ′ (x) = 0, когда 1 − x4 / 3 = 0,1 − x4 / 3 = 0. Следовательно, f ′ (x) = 0f ′ (x) = 0 при x = ± 1.x = ± 1. Производная f ′ (x) f ′ (x) не определена при x = 0.x = 0. Следовательно, у нас есть три критических точки: x = 0, x = 0, x = 1, x = 1 и x = −1.x = −1. Следовательно, разделим интервал (−∞, ∞) (- ∞, ∞) на меньшие интервалы (−∞, −1), (- 1,0), (0,1), (- ∞, −1), (−1,0), (0,1) и (1, ∞). (1, ∞).

    Шаг 2: Поскольку f′f ′ непрерывен на каждом подынтервале, достаточно выбрать контрольную точку xx в каждом из интервалов шага 11 и определить знак f′f ′ в каждой из этих точек.Точки x = −2, x = −12, x = 12 и x = 2x = −2, x = −12, x = 12 и x = 2 являются контрольными точками для этих интервалов.

    Интервал Контрольная точка Знак f ′ (x) = 5 (1 − x4 / 3) 3×2 / 3f ′ (x) = 5 (1 − x4 / 3) 3×2 / 3 в контрольной точке Заключение
    (−∞, −1) (- ∞, −1) х = -2 х = -2 (+) (-) + = – (+) (-) + = – ff уменьшается.
    (−1,0) (- 1,0) х = -12х = -12 (+) (+) + = + (+) (+) + = + ff увеличивается.
    (0,1) (0,1) х = 12х = 12 (+) (+) + = + (+) (+) + = + ff увеличивается.
    (1, ∞) (1, ∞) х = 2х = 2 (+) (-) + = – (+) (-) + = – ff уменьшается.

    Шаг 3: Поскольку ff убывает на интервале (−∞, −1) (- ∞, −1) и увеличивается на интервале (−1,0), (- 1,0), ff имеет локальный минимум при x = −1.x = −1. Поскольку ff возрастает на интервале (−1,0) (- 1,0) и интервале (0,1), (0,1), ff не имеет локального экстремума при x = 0.х = 0. Поскольку ff возрастает на интервале (0,1) (0,1) и убывает на интервале (1, ∞), f (1, ∞), f имеет локальный максимум при x = 1.x = 1. Аналитические результаты согласуются со следующим графиком.

    Фигура 4,33 Функция f имеет локальный минимум при x = −1x = −1 и локальный максимум при x = 1.x = 1.

    Контрольно-пропускной пункт 4,17

    Используйте тест первой производной, чтобы найти все локальные экстремумы для f (x) = x − 13. F (x) = x − 13.

    Вогнутость и точки перегиба

    Теперь мы знаем, как определить, где функция увеличивается или уменьшается.Однако есть еще одна проблема, которую следует учитывать в отношении формы графика функции. Если график изгибается, изгибается ли он вверх или вниз? Это понятие называется вогнутостью функции.

    На рис. 4.34 (a) показана функция ff с графиком, изгибающимся вверх. По мере увеличения xx наклон касательной увеличивается. Таким образом, поскольку производная увеличивается с увеличением xx, f′f ′ является возрастающей функцией. Мы говорим, что эта функция ff вогнута вверх. На рис. 4.34 (b) показана функция ff, которая изгибается вниз.По мере увеличения xx наклон касательной уменьшается. Поскольку производная убывает с увеличением xx, f′f ′ – убывающая функция. Мы говорим, что эта функция ff вогнута вниз.

    Определение

    Пусть ff – функция, дифференцируемая на открытом интервале I.I. Если f′f ′ возрастает над I, I, мы говорим, что ff вогнута вверх над I.I. Если f′f ′ убывает над I, I, мы говорим, что ff вогнута вниз над I.I.

    Фигура 4,34 (a), (c) Поскольку f′f ′ возрастает на интервале (a, b), (a, b), мы говорим, что ff вогнутая вверх на интервале (a, b).(а, б). (b), (d) Поскольку f′f ′ убывает на интервале (a, b), (a, b), мы говорим, что ff вогнутая вниз на (a, b). (a, b).

    В общем, не имея графика функции f, f, как мы можем определить ее вогнутость? По определению функция ff вогнута вверх, если f′f ′ возрастает. Из следствия 3,3 мы знаем, что если f′f ′ – дифференцируемая функция, то f′f ′ возрастает, если ее производная f ″ (x)> 0. f ″ (x)> 0. Следовательно, дважды дифференцируемая функция ff будет вогнутой, когда f ″ (x)> 0.f ″ (x)> 0. Точно так же функция ff вогнута вниз, если f′f ′ убывает. Мы знаем, что дифференцируемая функция f′f ′ убывает, если ее производная f ″ (x) <0. f ″ (x) <0. Следовательно, дважды дифференцируемая функция ff вогнута вниз, когда f ″ (x) <0. f ″ (x) <0. Применение этой логики известно как тест на вогнутость.

    Теорема 4.10

    Тест на вогнутость

    Пусть ff – функция, дважды дифференцируемая на интервале I.I.

    1. Если f ″ (x)> 0f ″ (x)> 0 для всех x∈I, x∈I, то ff вогнута вверх над I.I.
    2. Если f ″ (x) <0f ″ (x) <0 для всех x∈I, x∈I, то ff вогнута вниз над I.I.

    Мы заключаем, что мы можем определить вогнутость функции ff, глядя на вторую производную f.f. Кроме того, мы видим, что функция ff может переключать вогнутость (рисунок 4.35). Однако непрерывная функция может переключать вогнутость только в точке xx, если f ″ (x) = 0f ″ (x) = 0 или f ″ (x) f ″ (x) не определено. Следовательно, чтобы определить интервалы, в которых функция ff вогнута вверх и вогнута вниз, мы ищем те значения xx, где f ″ (x) = 0f ″ (x) = 0 или f ″ (x) f ″ (x) равно неопределенный.Когда мы определили эти точки, мы разделим область определения ff на меньшие интервалы и определим знак f ″ f ″ для каждого из этих меньших интервалов. Если f ″ f ″ меняет знак при прохождении через точку x, x, то ff меняет вогнутость. Важно помнить, что функция ff не может изменять вогнутость в точке xx, даже если f ″ (x) = 0f ″ (x) = 0 или f ″ (x) f ″ (x) не определено. Если, однако, ff действительно меняет вогнутость в точке aa и ff непрерывен в a, a, мы говорим, что точка (a, f (a)) (a, f (a)) является точкой перегиба f.f.

    Определение

    Если ff непрерывен в aa, а ff изменяет вогнутость в a, a, точка (a, f (a)) (a, f (a)) является точкой перегиба f.f.

    Фигура 4,35 Поскольку f ″ (x)> 0f ″ (x)> 0 для x a, x> a, функция ff вогнута вниз на интервале (a, ∞). (A, ∞). Точка (a, f (a)) (a, f (a)) является точкой перегиба f.f.

    Пример 4.19

    Испытание на вогнутость

    Для функции f (x) = x3−6×2 + 9x + 30, f (x) = x3−6×2 + 9x + 30 определите все интервалы, где ff вогнута вверх, и все интервалы, где ff вогнута вниз. Перечислите все точки перегиба для f.f. Используйте графическую утилиту, чтобы подтвердить свои результаты.

    Решение

    Чтобы определить вогнутость, нам нужно найти вторую производную f ″ (x) .f ″ (x). Первая производная равна f ′ (x) = 3×2−12x + 9, f ′ (x) = 3×2−12x + 9, поэтому вторая производная равна f ″ (x) = 6x − 12.f ″ (x) = 6x − 12. Если функция изменяет вогнутость, это происходит либо когда f ″ (x) = 0f ″ (x) = 0, либо f ″ (x) f ″ (x) не определено. Поскольку f ″ f ″ определено для всех действительных чисел x, x, нам нужно только найти, где f ″ (x) = 0. f ″ (x) = 0. Решая уравнение 6x − 12 = 0,6x − 12 = 0, мы видим, что x = 2x = 2 – единственное место, где ff может изменить вогнутость. Теперь мы проверяем точки на интервалах (−∞, 2) (- ∞, 2) и (2, ∞) (2, ∞), чтобы определить вогнутость f.f. Точки x = 0x = 0 и x = 3x = 3 являются контрольными точками для этих интервалов.

    Интервал Контрольная точка Знак f ″ (x) = 6x − 12f ″ (x) = 6x − 12 в контрольной точке Заключение
    (−∞, 2) (- ∞, 2) х = 0х = 0 −− ff вогнутая вниз
    (2, ∞) (2, ∞) х = 3х = 3 ++ ff вогнутый вверх.

    Мы заключаем, что ff вогнута вниз на интервале (−∞, 2) (- ∞, 2) и вогнута вверх на интервале (2, ∞). (2, ∞). Поскольку ff изменяет вогнутость при x = 2, x = 2, точка (2, f (2)) = (2,32) (2, f (2)) = (2,32) является точкой перегиба. Рисунок 4.36 подтверждает аналитические результаты.

    Фигура 4,36 Данная функция имеет точку перегиба в (2,32) (2,32), где график меняет вогнутость.

    Контрольно-пропускной пункт 4,18

    Для f (x) = – x3 + 32×2 + 18x, f (x) = – x3 + 32×2 + 18x найдите все интервалы, где ff вогнута вверх, и все интервалы, где ff вогнута вниз.

    Теперь мы суммируем в таблице 4.1 информацию, которую первая и вторая производные функции ff предоставляют о графике f, f, и проиллюстрируем эту информацию на рисунке 4.37.

    Знак f′f ′ Знак f ″ f ″ Ff увеличивается или уменьшается? Вогнутость
    Положительный Положительно Увеличение Вогнутый вверх
    Положительный отрицательный Увеличение Вогнутая вниз
    Отрицательный Положительно Уменьшение Вогнутый вверх
    Отрицательный отрицательный Уменьшение Вогнутая вниз

    Таблица 4.1 Что производные говорят нам о графиках

    Фигура 4,37 Рассмотрим дважды дифференцируемую функцию ff на открытом интервале И.И. Если f ′ (x)> 0f ′ (x)> 0 для всех x∈I, x∈I, функция возрастает по I.I. Если f ′ (x) <0f ′ (x) <0 для всех x∈I, x∈I, функция убывает по I.I. Если f ″ (x)> 0f ″ (x)> 0 для всех x∈I, x∈I, функция вогнута вверх. Если f ″ (x) <0f ″ (x) <0 для всех x∈I, x∈I, функция вогнута вниз на I.I.

    Тест второй производной

    Тест первой производной предоставляет аналитический инструмент для поиска локальных экстремумов, но вторая производная также может использоваться для определения экстремальных значений.Иногда использование второй производной может быть более простым методом, чем использование первой производной.

    Мы знаем, что если у непрерывной функции есть локальные экстремумы, они должны возникать в критической точке. Однако функция не обязательно должна иметь локальные экстремумы в критической точке. Здесь мы исследуем, как можно использовать тест второй производной, чтобы определить, имеет ли функция локальный экстремум в критической точке. Пусть ff – дважды дифференцируемая функция такая, что f ′ (a) = 0f ′ (a) = 0 и f ″ f ″ непрерывна на открытом интервале II, содержащем a.а. Предположим, что f ″ (a) <0. f ″ (a) <0. Поскольку f ″ f ″ непрерывна над I, I, f ″ (x) <0f ″ (x) <0 для всех x∈Ix∈I (рис. 4.38). Тогда по следствию 3,3 f′f ′ - убывающая функция над I.I. Поскольку f ′ (a) = 0, f ′ (a) = 0, мы заключаем, что для всех x∈I, f ′ (x)> 0x∈I, f ′ (x)> 0, если x ax> a. Следовательно, по тесту первой производной ff имеет локальный максимум при x = a.x = a. С другой стороны, предположим, что существует точка bb такая, что f ′ (b) = 0f ′ (b) = 0, но f ″ (b)> 0. f ″ (b)> 0. Поскольку f ″ f ″ непрерывна на открытом интервале II, содержащем b, b, то f ″ (x)> 0f ″ (x)> 0 для всех x∈Ix∈I (рисунок 4.38). Тогда по следствию 3 f′3, f ′ – возрастающая функция над I.I. Поскольку f ′ (b) = 0, f ′ (b) = 0, заключаем, что для всех x∈I, x∈I, f ′ (x) <0f ′ (x) <0, если x 0f ′ (x)> 0, если x> bx> b. Следовательно, по тесту первой производной ff имеет локальный минимум при x = b.x = b.

    Фигура 4,38 Рассмотрим дважды дифференцируемую функцию ff такую, что f ″ f ″ непрерывна. Поскольку f ′ (a) = 0f ′ (a) = 0 и f ″ (a) <0, f ″ (a) <0, существует интервал II, содержащий aa, такой, что для всех xx в I, I, ff равно увеличивается, если x a.х> а. В результате ff имеет локальный максимум при x = a.x = a. Поскольку f ′ (b) = 0f ′ (b) = 0 и f ″ (b)> 0, f ″ (b)> 0, существует интервал II, содержащий bb, такой, что для всех xx в I, I, ff равно уменьшается, если x bx> b. В результате ff имеет локальный минимум при x = b.x = b.

    Теорема 4.11

    Второй производный тест

    Предположим, что f ′ (c) = 0, f ″ f ′ (c) = 0, f ″ непрерывно на интервале, содержащем c.c.

    1. Если f ″ (c)> 0, f ″ (c)> 0, то ff имеет локальный минимум в c.c.
    2. Если f ″ (c) <0, f ″ (c) <0, то ff имеет локальный максимум в c.c.
    3. Если f ″ (c) = 0, f ″ (c) = 0, то проверка не дает результатов.

    Обратите внимание, что для случая iii. когда f ″ (c) = 0, f ″ (c) = 0, тогда ff может иметь локальный максимум, локальный минимум или ни один из них в c.c. Например, функции f (x) = x3, f (x) = x3, f (x) = x4, f (x) = x4 и f (x) = – x4f (x) = – x4 все имеют критические указывает на x = 0.x = 0. В каждом случае вторая производная равна нулю при x = 0.x = 0. Однако функция f (x) = x4f (x) = x4 имеет локальный минимум при x = 0x = 0, тогда как функция f (x) = – x4f (x) = – x4 имеет локальный максимум в x, x, а функция f (x) = x3f (x) = x3 не имеет локального экстремума при x = 0.х = 0.

    Давайте теперь посмотрим, как использовать тест второй производной, чтобы определить, имеет ли ff локальный максимум или локальный минимум в критической точке cc, где f ′ (c) = 0. f ′ (c) = 0.

    Пример 4.20

    Использование теста второй производной

    Используйте вторую производную, чтобы найти расположение всех локальных экстремумов для f (x) = x5−5×3.f (x) = x5−5×3.

    Решение

    Чтобы применить тест второй производной, нам сначала нужно найти критические точки cc, где f ′ (c) = 0.f ′ (c) = 0. Производная равна f ′ (x) = 5×4−15×2.f ′ (x) = 5×4−15×2. Следовательно, f ′ (x) = 5×4−15×2 = 5×2 (x2−3) = 0f ′ (x) = 5×4−15×2 = 5×2 (x2−3) = 0, когда x = 0, ± 3.x = 0, ± 3.

    Чтобы определить, есть ли у ff локальные экстремумы в любой из этих точек, нам нужно оценить знак f ″ f ″ в этих точках. Вторая производная –

    f ″ (x) = 20×3−30x = 10x (2×2−3). f ″ (x) = 20×3−30x = 10x (2×2−3).

    В следующей таблице мы оцениваем вторую производную в каждой из критических точек и используем тест второй производной, чтобы определить, имеет ли ff локальный максимум или локальный минимум в любой из этих точек.

    х х f ″ (x) f ″ (x) Заключение
    −3−3 −303−303 Местный максимум
    00 00 Тест второй производной безрезультатно
    33 303303 Местный минимум

    Используя проверку второй производной, мы заключаем, что ff имеет локальный максимум при x = −3x = −3, а ff имеет локальный минимум при x = 3.х = 3. Тест второй производной не дает результатов при x = 0.x = 0. Чтобы определить, есть ли у ff локальные экстремумы при x = 0, x = 0, мы применяем тест первой производной. Чтобы оценить знак f ′ (x) = 5×2 (x2−3) f ′ (x) = 5×2 (x2−3) для x∈ (−3,0) x∈ (−3,0) и x∈ ( 0,3), x∈ (0,3), пусть x = −1x = −1 и x = 1x = 1 – две контрольные точки. Поскольку f ′ (- 1) <0f ′ (- 1) <0 и f ′ (1) <0, f ′ (1) <0, мы заключаем, что ff убывает на обоих интервалах и, следовательно, ff не имеет локальные экстремумы при x = 0x = 0, как показано на следующем графике.

    Фигура 4.39 Функция ff имеет локальный максимум при x = −3x = −3 и локальный минимум при x = 3x = 3.

    Контрольно-пропускной пункт 4,19

    Рассмотрим функцию f (x) = x3− (32) x2−18x.f (x) = x3− (32) x2−18x. Точки c = 3, −2c = 3, −2 удовлетворяют условию f ′ (c) = 0. f ′ (c) = 0. Используйте тест второй производной, чтобы определить, имеет ли ff локальный максимум или локальный минимум в этих точках.

    Теперь мы разработали инструменты, необходимые для определения того, где функция увеличивается и уменьшается, а также получили понимание основной формы графика.В следующем разделе мы обсудим, что происходит с функцией при x → ± ∞.x → ± ∞. На данный момент у нас есть достаточно инструментов для создания точных графиков большого количества функций.

    Раздел 4.5 Упражнения

    194 .

    Если cc является критической точкой для f (x), f (x), когда нет локального максимума или минимума в c? C? Объяснять.

    195 .

    Для функции y = x3, y = x3, является ли x = 0x = 0 точкой перегиба и локальным максимумом / минимумом?

    196 .

    Является ли x = 0x = 0 точкой перегиба для функции y = x3, y = x3?

    197 .

    Может ли точка cc быть одновременно точкой перегиба и локальным экстремумом дважды дифференцируемой функции?

    198 .

    Зачем нужна непрерывность для первого теста производной? Придумайте пример.

    199 .

    Объясните, должна ли функция вогнутого вниз пересекать y = 0y = 0 для некоторого значения x.x.

    200 .

    Объясните, может ли многочлен степени 22 иметь точку перегиба.

    Для следующих упражнений проанализируйте графики f ‘, f’, затем перечислите все интервалы, в которых ff увеличивается или уменьшается.

    202 . 204 .

    Для следующих упражнений проанализируйте графики f ′, f ′, затем перечислите все интервалы, где

    1. ff увеличивается и уменьшается и
    2. расположены минимумы и максимумы.
    206 . 208 . 210 .

    Для следующих упражнений проанализируйте графики f ‘, f’, затем перечислите все точки перегиба и интервалы ff, которые вогнуты вверх и вогнуты вниз.

    212 . 214 .

    Для следующих упражнений нарисуйте граф, удовлетворяющий заданным спецификациям для области xϵ [−3,3].хϵ [-3,3]. Функция не обязательно должна быть непрерывной или дифференцируемой.

    216 .

    f (x)> 0, f ′ (x)> 0f (x)> 0, f ′ (x)> 0 над x> 1, −3 1, −3 217 .

    f ′ (x)> 0f ′ (x)> 0 над x> 2, −3 2, −3 218 .

    f ″ (x) <0f ″ (x) <0 сверх −1 0, −3 0, −3 219 .

    Существует локальный максимум при x = 2, x = 2, локальный минимум при x = 1, x = 1, и график не является ни вогнутым вверх, ни вогнутым вниз.

    220 .

    Имеются локальные максимумы при x = ± 1, x = ± 1, функция вогнута вверх для всех x, x, и функция остается положительной для всех x.x.

    Для следующих упражнений определите

    1. интервалов, где ff увеличивается или уменьшается и
    2. локальных минимумов и максимумов ф.ф.
    221 .

    f (x) = sinx + sin3xf (x) = sinx + sin3x над −π

    Для следующих упражнений определите a.интервалы, где ff вогнута вверх или вогнута вниз, и b. точки перегиба ф.ф.

    223 .

    f (x) = x3−4×2 + x + 2f (x) = x3−4×2 + x + 2.

    Для следующих упражнений определите

    1. интервалы увеличения или уменьшения ff,
    2. локальных минимумов и максимумов f, f,
    3. интервалов, где ff является вогнутым вверх и вогнутым вниз, и
    4. точки перегиба ф.ф.
    225 .

    f (х) = x3−6x2f (x) = x3−6×2

    226 .

    f (x) = x4−6x3f (x) = x4−6×3

    227 .

    f (x) = x11−6x10f (x) = x11−6×10

    228 .

    f (x) = x + x2 − x3 f (x) = x + x2 − x3

    Для следующих упражнений определите

    1. интервалы увеличения или уменьшения ff,
    2. локальных минимумов и максимумов f, f,
    3. интервалов, где ff является вогнутым вверх и вогнутым вниз, и
    4. точки перегиба ф.ф. Нарисуйте кривую, а затем с помощью калькулятора сравните свой ответ. Если вы не можете определить точный ответ аналитически, воспользуйтесь калькулятором.
    231 .

    [T] f (x) = sin (πx) −cos (πx) f (x) = sin (πx) −cos (πx) над x = [- 1,1] x = [- 1,1 ]

    232 .

    [T] f (x) = x + sin (2x) f (x) = x + sin (2x) over x = [- π2, π2] x = [- π2, π2]

    233 .

    [T] f (x) = sinx + tanxf (x) = sinx + tanx над (−π2, π2) (- π2, π2)

    234 .

    [T] f (x) = (x − 2) 2 (x − 4) 2f (x) = (x − 2) 2 (x − 4) 2

    235 .

    [T] f (x) = 11 − x, x ≠ 1f (x) = 11 − x, x ≠ 1

    236 .

    [T] f (x) = sinxxf (x) = sinxx над x = x = [2π, 0) ∪ (0,2π] [2π, 0) ∪ (0,2π]

    237 .

    f (x) = sin (x) exf (x) = sin (x) ex над x = [- π, π] x = [- π, π]

    238 .

    f (x) = lnxx, x> 0 f (x) = lnxx, x> 0

    239 .

    f (x) = 14x + 1x, x> 0 f (x) = 14x + 1x, x> 0

    240 .

    f (x) = exx, x ≠ 0 f (x) = exx, x ≠ 0

    Для следующих упражнений интерпретируйте предложения в терминах f, f ′ и f ″ .f, f ′ и f ″.

    241 .

    Население растет медленнее. Здесь ff – население.

    242 .

    Велосипед ускоряется быстрее, но машина едет быстрее.Здесь f = f = положение велосипеда минус положение автомобиля.

    243 .

    Самолет плавно приземляется. Здесь ff – высота самолета.

    244 .

    Цены на акции на пике. Здесь ff – цена акции.

    245 .

    Экономика набирает обороты. Здесь ff – это показатель экономики, например ВВП.

    Для следующих упражнений рассмотрим многочлен третьей степени f (x), f (x), который обладает свойствами f ′ (1) = 0, f ′ (3) = 0. f ′ (1) = 0, f ′ (3) = 0. Определите, являются ли следующие утверждения истинными или ложными .Обосновать ответ.

    246 .

    f (x) = 0f (x) = 0 для некоторых 1≤x≤31≤x≤3

    247 .

    f ″ (x) = 0f ″ (x) = 0 для некоторых 1≤x≤31≤x≤3

    248 .

    Абсолютного максимума при x = 3x = 3 не существует

    249 .

    Если f (x) f (x) имеет три корня, то у нее 11 точек перегиба.

    250 .

    Если f (x) f (x) имеет одну точку перегиба, то она имеет три действительных корня.

    Исчисление III – Интерпретация частных производных

    Показать уведомление для мобильных устройств Показать все заметки Скрыть все заметки

    Похоже, вы используете устройство с “узкой” шириной экрана ( i.е. вы, вероятно, пользуетесь мобильным телефоном). Из-за характера математики на этом сайте лучше всего просматривать в ландшафтном режиме. Если ваше устройство не находится в альбомном режиме, многие уравнения будут отображаться сбоку от вашего устройства (вы должны иметь возможность прокручивать, чтобы увидеть их), а некоторые элементы меню будут обрезаны из-за узкой ширины экрана.

    Раздел 2-3: Интерпретация частичных производных инструментов

    Это довольно короткий раздел, и здесь мы можем признать, что две основные интерпретации производных функций одной переменной все еще справедливы для частных производных, с небольшими изменениями, конечно, с учетом того факта, что теперь у нас есть более одного Переменная.

    Первая интерпретация, которую мы уже видели, является наиболее важной из двух. Как и в случае с функциями отдельных переменных, частные производные представляют собой скорость изменения функций при изменении переменных. Как мы видели в предыдущем разделе, \ ({f_x} \ left ({x, y} \ right) \) представляет скорость изменения функции \ (f \ left ({x, y} \ right) \) поскольку мы меняем \ (x \) и фиксируем \ (y \), а \ ({f_y} \ left ({x, y} \ right) \) представляет скорость изменения \ (f \ left ({x, y} \ right) \), когда мы меняем \ (y \) и фиксируем \ (x \). 4}}} \ hspace {0.5in} \ Rightarrow \ hspace {0.5in} {f_y} \ left ({2,5} \ right) = – \ frac {{12}} {{625}}

    Здесь частная производная по \ (y \ ) отрицательно, поэтому функция убывает в точке \ (\ left ({2,5} \ right) \), когда мы меняем \ (y \) и фиксируем \ (x \).

    Обратите внимание, что функция вполне может увеличиваться при фиксированном \ (y \) и уменьшаться при фиксированном \ (x \) в точке, как показано в этом примере. Чтобы увидеть хороший пример этого, взгляните на следующий график.

    Это график гиперболического параболоида, и в начале координат мы можем видеть, что если мы продвинемся вдоль оси \ (y \), график будет увеличиваться, а если мы будем двигаться вдоль оси \ (x \), график будет уменьшается. Таким образом, вполне возможно иметь график, как увеличивающийся, так и убывающий в точке, в зависимости от направления, в котором мы движемся. Никогда не следует ожидать, что функция будет вести себя точно так же в какой-то момент при изменении каждой переменной.

    Следующая интерпретация была одной из стандартных интерпретаций в классе «Исчисление I».Мы знаем из класса Исчисления I, что \ (f ‘\ left (a \ right) \) представляет наклон касательной к \ (y = f \ left (x \ right) \) в точке \ (x = a \ ). Итак, \ ({f_x} \ left ({a, b} \ right) \) и \ ({f_y} \ left ({a, b} \ right) \) также представляют собой наклоны касательных прямых. Разница здесь в функциях, к которым они относятся.

    Частные производные – это наклон трасс. Частная производная \ ({f_x} \ left ({a, b} \ right) \) – это наклон следа \ (f \ left ({x, y} \ right) \) для плоскости \ (y = b \) в точке \ (\ left ({a, b} \ right) \).2} \) в точке \ (\ left ({1,2} \ right) \). Показать решение

    Мы нарисовали трассы для плоскостей \ (x = 1 \) и \ (y = 2 \) в предыдущем разделе, и это две трассы для этой точки. Для справки здесь представлены графики трасс.

    Затем нам потребуются две частные производные, чтобы мы могли получить наклоны.

    \ [{f_x} \ left ({x, y} \ right) = – 8x \ hspace {0,5 дюйма} {f_y} \ left ({x, y} \ right) = – 2y \]

    Чтобы получить наклоны, все, что нам нужно сделать, это оценить частные производные в рассматриваемой точке.2} \) для плоскости \ (x = 1 \) имеет наклон -4.

    Наконец, давайте кратко поговорим о получении уравнений касательной. Напомним, что уравнение линии в трехмерном пространстве задается векторным уравнением. Кроме того, чтобы получить уравнение, нам нужны точка на прямой и вектор, параллельный этой линии.

    Дело простое. Поскольку мы знаем координаты точки \ (x \) – \ (y \), все, что нам нужно сделать, это вставить это в уравнение, чтобы получить точку.Итак, точка будет,

    \ [\ left ({a, b, f \ left ({a, b} \ right)} \ right) \]

    Параллельный (или касательный) вектор также прост. Мы можем записать уравнение поверхности в виде вектор-функции следующим образом:

    \ [\ vec r \ left ({x, y} \ right) = \ left \ langle {x, y, z} \ right \ rangle = \ left \ langle {x, y, f \ left ({x, y } \ right)} \ right \ rangle \]

    Мы знаем, что если у нас есть векторная функция одной переменной, мы можем получить касательный вектор, дифференцируя векторную функцию.То же самое и здесь. Если мы продифференцируем по \ (x \), мы получим касательный вектор к следам для плоскости \ (y = b \) (, т.е. для фиксированного \ (y \)), и если мы будем дифференцировать по \ ( y \) получим касательный вектор к следам для плоскости \ (x = a \) (или фиксированного \ (x \)).

    Итак, вот касательный вектор для следов с фиксированным \ (y \).

    \ [{\ vec r_x} \ left ({x, y} \ right) = \ left \ langle {1,0, {f_x} \ left ({x, y} \ right)} \ right \ rangle \]

    Мы дифференцировали каждый компонент по \ (x \).Следовательно, первый компонент становится 1, а второй становится нулем, потому что мы рассматриваем \ (y \) как константу, когда мы дифференцируем по \ (x \). Третий компонент – это просто частная производная функции по \ (x \).

    Для трасс с фиксированным \ (x \) касательный вектор равен

    \ [{\ vec r_y} \ left ({x, y} \ right) = \ left \ langle {0,1, {f_y} \ left ({x, y} \ right)} \ right \ rangle \]

    Уравнение касательной к следам при фиксированном \ (y \) тогда равно

    \ [\ vec r \ left (t \ right) = \ left \ langle {a, b, f \ left ({a, b} \ right)} \ right \ rangle + t \ left \ langle {1,0, {f_x} \ left ({a, b} \ right)} \ right \ rangle \]

    , а касательная к следам с фиксированным \ (x \) равна

    \ [\ vec r \ left (t \ right) = \ left \ langle {a, b, f \ left ({a, b} \ right)} \ right \ rangle + t \ left \ langle {0,1, {f_y} \ left ({a, b} \ right)} \ right \ rangle \] Пример 3 Запишите векторные уравнения касательных к следам к \ (z = 10 – 4 {x ^ 2} – {y ^ 2} \) в точке \ (\ left ({1,2} \ right ) \).Показать решение

    На самом деле здесь не так уж много общего, кроме как подставить значения и функции в приведенные выше формулы. Мы уже вычислили производные и их значения в \ (\ left ({1,2} \ right) \) в предыдущем примере, и точка на каждой трассе равна

    . \ [\ left ({1,2, f \ left ({1,2} \ right)} \ right) = \ left ({1,2,2} \ right) \]

    Вот уравнение касательной к следу для плоскости \ (y = 2 \).

    \ [\ vec r \ left (t \ right) = \ left \ langle {1,2,2} \ right \ rangle + t \ left \ langle {1,0, – 8} \ right \ rangle = \ left \ langle {1 + t, 2,2 – 8t} \ right \ rangle \]

    Вот уравнение касательной к следу для плоскости \ (x = 1 \).

    \ [\ vec r \ left (t \ right) = \ left \ langle {1,2,2} \ right \ rangle + t \ left \ langle {0,1, – 4} \ right \ rangle = \ left \ langle {1,2 + t, 2 – 4t} \ right \ rangle \]

    Краткое введение в производные инструменты для машинного обучения | Майкл Грин

    Введение

    Если вы похожи на меня, то, вероятно, большую часть своей жизни использовали деривативы и выучили несколько правил их работы и поведения, не понимая, откуда все это взялось.В детстве мы узнаем некоторые из этих правил на раннем этапе, например, правило мощности, в котором мы знаем, что производная от равна 2x , что в более общей форме превращается в dxᵃ / dx = axᵃ⁻¹ . В принципе, это нормально, поскольку все правила можно легко запомнить и просмотреть в таблице. Обратной стороной этого является, конечно, то, что вы используете систему и формализм, которых вы принципиально не понимаете. Опять же, это не обязательно проблема, если вы сами не разрабатываете фреймворки машинного обучения на ежедневной основе, но, тем не менее, очень приятно знать, что происходит за кулисами.Я сам презираю черные ящики ◼. Итак, чтобы углубиться в это, я покажу вам, на чем все это основано. Для этого мы должны определить, что производная должна делать для вас. Вы знаете? Я уверен, что да, но на всякий случай нет;

    Производная – это непрерывное описание того, как функция изменяется при небольших изменениях одной или нескольких переменных.

    Мы рассмотрим многие аспекты этого утверждения. Например

    • Что значит маленький?
    • Что означает изменение?
    • Почему непрерывный?
    • Чем это полезно?

    Поехали!

    Эти термины обычно сбивают с толку многих, поскольку иногда их считают равнозначными, а во многих случаях они кажутся неотличимыми друг от друга.Однако это не так! Давайте начнем с определения частной производной, а затем перейдем к полной производной оттуда. Для этой цели я буду использовать мнимую функцию f (t, x, y) , где у нас есть три переменные: t , x и y . Частная производная отвечает на вопросы о том, как f изменяется (∂ f ), когда одна переменная изменяется на небольшую величину (∂ x ). В этой настройке все другие переменные считаются постоянными и статическими.Таким образом, частная производная обозначается ∂ f / ∂ x . Чтобы показать, что происходит, когда мы выполняем эту операцию, нам нужно сначала определить f как что-то. Допустим, это выглядит так: f (t, x, y) = txy 4 π / 3 , что, кстати, является объемом эллипсоида. Что ж, возможно, не так уж и случайно … В любом случае я выбрал другую параметризацию, чем обычно используется. На картинке ниже вы можете видеть сверху налево и направо сферу, сфероид и эллипсоид соответственно.В наших настройках мы можем выбрать размеры t = a, x = b, y = c .

    Автор Ag2gaeh – собственная работа, CC BY-SA 4.0, https://commons.wikimedia.org/w/index.php?curid=45585493

    Частная производная объема этих геометрических пространств тогда принимает вид ∂ f / ∂ x = 4 π 3 ty , где мы применили правило мощности. Как вы видите, t и y не были затронуты, поскольку мы предполагали, что они исправлены. Таким образом, на изображении выше мы моделируем то, что происходит с объемом, когда b расширяется или укорачивается на небольшую величину.Это отвечает на наш вопрос, действительно ли они независимы от x . Но что, если это не так? В этом случае нам нужна полная производная f по отношению к x , которая обозначается df / dx и определяется как

    , где вы можете видеть частную производную как часть общей. Итак, для наглядности давайте ограничим функцию ситуацией, когда t = x . Что тогда происходит с производной? Что ж, частная производная от предыдущего остается неизменной.но два других члена нам нужно вычислить. Первая часть становится (∂f / ∂t) (dt / dx) = 4π / 3 ⋅ xy ⋅ 1, а последняя часть превращается в

    Таким образом, теперь мы получаем

    , добавляя члены и подставляя t = x в последний шаг. Теперь, надеюсь, очевидно, что ∂f / ∂x ≠ df / dx и что вам нужно быть осторожным, прежде чем заявлять независимость между вашими переменными при выполнении ваших производных.

    ПОДОЖДИТЕ! Я слышу, как вы плачете, не могли бы мы просто произвести замену после того, как вычислили частную производную? В самом деле, вы могли бы и получили бы что-то, что не так в два раза, что может быть значительным.В основном вы получите следующее безумие

    Это потому, что то, что мы обычно ищем, действительно является полной производной, а не частичной. Однако вы, конечно, могли бы сделать замену до того, как вы вычислили частную производную. Это будет похоже на

    , где мы снова достигаем согласованности. Таким образом, вы не можете подставить зависимости в свою частную производную после того, как было вычислено!

    Давайте ненадолго вернемся к определению полной производной.Помните, что это выглядело так:

    для функции f (t, x, y) с тремя переменными. Теперь, если мы умножим это везде на dx , мы получим

    , что является выражением дифференциального взгляда на функцию f . В нем говорится, что очень небольшое изменение в f может быть определено как взвешенная сумма небольших изменений в его переменных, где веса являются частными производными функции по тем же переменным. В целом мы можем сформулировать это для функции q с переменными M , например,

    , что является гораздо более компактным и красивым способом рассмотрения.Явное написание терминов быстро становится утомительным. С другой стороны, мы также получаем компактный способ представления нашего определения полной производной. Снова придерживаемся функции q с ее переменными M .

    Значение δᵢₚ определяется как 1 везде, кроме i = p , и в этом случае мы определяем его как 0 . Я знаю, что это не очень традиционно, но это работает, поэтому я буду использовать дельта-функцию таким образом. Я делаю это потому, что

    , хотя и является правильным, не акцентирует внимание на частной производной интересующей переменной xₚ, но на самом деле это вопрос вкуса и совсем не важен для использования.

    Одним из, пожалуй, наиболее распространенных правил, используемых при вычислении аналитических производных финансовых инструментов, является правило цепочки. Математически это в основном означает следующее:

    , что не выглядит впечатляющим, но не позволяйте своей простоте вводить вас в заблуждение. Это бесподобная рабочая лошадка в аналитическом мире градиентов. Помните, что g (x) может быть любым в этой настройке. Так и x , если на то пошло. Таким образом, это правило применимо ко всему, что касается градиентов.

    Небольшое примечание относительно именования.«Цепное правило» на самом деле существует в вероятности также под названием «Цепное правило вероятности» или «Общее правило продукта». Я считаю последнее более естественным. В любом случае это правило утверждает следующее:

    , где p – функция вероятности для событий x и y . Это правило может быть далее обобщено на n переменных, повторяя это правило. См. Следующий пример:

    Можно простить вас за то, что вы поверите, что порядок переменных каким-то образом имеет значение при применении этого правила, но, конечно, это не так, поскольку все, что мы делаем, – это разбиваем вероятностное пространство на более мелкие независимые участки.Таким образом, в более компактном формате мы можем выразить это общее правило следующим образом:

    , где мы использовали n общих переменных, представляющих наш вероятностный ландшафт. Теперь о причине, по которой я поднял это.

    Цепное правило вероятности не имеет ничего общего с цепным правилом исчисления.

    Поэтому не забывайте всегда думать о контексте, если вы слышите, как кто-то отказывается от «правила цепочки», поскольку без контекста это довольно неоднозначно.

    В этом разделе я проведу вас через простой многослойный персептрон и построение алгоритма обратного распространения ошибки.Есть много способов получить это, но я начну с подхода к минимизации ошибок, который в основном описывает соответствие нейронной сети f ( x , θ ) отклонением от известная цель y . Архитектура, которую мы будем решать, показана на изображении ниже, где у нас есть два скрытых слоя. Мы придерживаемся этого для простоты. Мы также будем использовать только один результат вместо нескольких, но он легко поддается обобщению.

    Иллюстрация простой двухуровневой нейронной сети с прямой связью.

    Вместо того, чтобы представлять нашу сеть графически, мы сделаем здесь более формальное представление, в котором функциональная форма будет указана математически. В основном функциональная форма будет иметь вид

    , где жирным шрифтом обозначены векторы. Функция φ (s) = 1 / (1 + exp ( a s)) представляет собой сигмовидную функцию активации с гиперпараметром a , который мы не будем настраивать или о котором не будем заботиться в этом введении.Небольшое замечание, игнорирование параметра a здесь действительно глупо, поскольку это коренным образом изменит обучение этой сети. Единственная причина, по которой я позволяю себе это делать, заключается в том, что сейчас это выходит за рамки возможностей охватить это.

    Чтобы обучить нейронную сеть, нам нужно обновить параметры в соответствии с тем, насколько они влияют на ошибку, которую мы видим. Эту ошибку можно определить так для проблемы, подобной регрессии, для точки данных ( x ₜ, yₜ).

    Если мы посмотрим на второй последний слой, мы просто обновим параметры в соответствии со следующим правилом

    для каждой новой точки данных.Это называется стохастическим градиентным спуском (SGD). Вы можете много прочитать об этом во многих местах, поэтому я не буду углубляться в это здесь. Достаточно сказать, что этот процесс может повторяться для каждого параметра на каждом уровне. Таким образом, печально известный алгоритм обратного распространения ошибки – это просто приложение для обновления ваших параметров с помощью частной производной ошибки по этому самому параметру. Сделайте частные производные для себя сейчас и посмотрите, насколько легко вы сможете их вывести. Небольшой трюк, который вы можете использовать, – это понять, что φ ‘(s) = φ (s) (1-φ (s)) , где я использовал простое обозначение для производной.Вот хороший учебник о том, как сделать это численно.

    • Полная производная и частная производная взаимосвязаны, но иногда фундаментально различаются.
    • Все ограничения и подстановки переменных должны быть выполнены до вычисления частной производной.
    • Частная производная игнорирует неявные зависимости.
    • Полная производная учитывает все зависимости.
    • Многие волшебные рецепты, такие как алгоритм обратного распространения ошибки, обычно исходят из довольно простых идей, и их выполнение для себя действительно поучительно и полезно.

    Справка в Интернете – Справка по Origin

    FitRef-PartDerivPara

    Итерационная процедура алгоритма L-M требует вычисления частных производных функций аппроксимации для достижения наилучшего соответствия. Поэтому, когда вы определяете свою пользовательскую функцию, указание частных производных может сократить время, необходимое для выполнения набора итераций.

    Если вы хотите указать частные производные, обратите внимание, что вы должны указать производные функции зависимой переменной в терминах подгоняемых параметров.

    Если вы не указали частные производные, Origin будет решать частные производные функций аппроксимации числовыми методами.

    Примечания : Однако, учитывая значительно более высокую скорость подгонки, достигаемую с помощью Origin C, указание частных производных может не потребоваться.
    • Указание частных производных поддерживается только при определении определяемой пользователем функции
    • Указание частных производных поддерживается только для двухфункциональной формы аппроксимирующей функции.
      • Происхождение C
      • Labtalk Script (Не рекомендуется использовать)

    Как установить параметры частной производной

    Использование конструктора функций

    1. На странице «Имя и тип» конструктора функций подгонки установите для Тип функции значение Origin C или Labtalk Script
    2. Продолжайте переходить на страницу тела функции, не забудьте установить флажок Use Derivatives .Затем нажмите кнопку Open Code Builder .
    3. Определите функцию и уравнения в частных производных в теле функции аппроксимации.

    Использование диспетчера функций

    В органайзере функций примерки

    1. Выберите Функциональная форма как «Y-Script» или Origin C
    2. Установите флажок Производная
    3. Нажмите кнопку Open Code Builder рядом с полем Function
    4. Определите функцию и уравнения в частных производных в теле функции аппроксимации

    Определение частной производной

    По происхождению C

    Если вы выбрали определение функции с помощью Origin C, переменные для частных производных будут предварительно определены в Code Builder.Вам просто нужно ввести для них выражение.

    Автор Labtalk

    Если вы хотите определить свою функцию с помощью LabTalk, выберите Y-Script в Fitting Function Organizer или Labtalk Script в Fitting Function Builder, используйте d_VarName для представления производных переменных.

    Однако Origin C проще в использовании, потому что он позволяет пользователю компилировать и оценивать функции до того, как они действительно начнут его использовать. Теперь мы не рекомендуем указывать частную производную с помощью Labtalk.

    Для того же примера, что и в руководстве в разделе выше

    Функция:

    Labtalk может определить функцию и частные производные, как показано ниже.

     у = р1 + р2 * х + ехр (р3 * х);
    dy_p1 = 1;
    dy_p2 = х;
    dy_p3 = х * ехр (р3 * х); 

    Высшие производные

    Когда мы берем производную функции, мы получаем другую функцию.Мы будем называть эту новую функцию первой производной по причинам, которые, надеюсь, станут понятны со временем. К настоящему времени вы должны быть довольны идеей, что производная функции – это другая функция, которая даст нам наклон исходной функции для любого значения x , для которого определена исходная функция. Поскольку первая производная сама по себе является функцией, понятно, что мы должны иметь возможность построить график этой функции. Также кажется разумным предположить, что мы должны иметь возможность взять его производную.Поскольку эта производная была бы производной производной, мы называем ее второй производной исходной функции. Давайте посмотрим на пример. Вот график функции ƒ ( x ) = 4 x 4 – 2 x 3 – 12 x 2 :


    График функции ƒ ( x ) = 4 x 4 -2 x 3 -12 x 2


    Если мы применим основные правила дифференцирования к этой функции, чтобы получить производную, мы получим:

    ƒ ′ ( x ) = 16 x 3 – 6 x 2 -24 x

    Обозначение, которое мы использовали для обозначения первой производной функции, – это обозначение Лагранжа , упомянутое в другом месте на этих страницах.В этой нотации один символ простое число («ʹ») помещается сразу после символа функции, чтобы указать, что это первая производная исходной функции. Чтобы указать вторую производную, мы помещаем два простых символа сразу после символа функции. Для обозначения третьей производной используются три символа простых чисел. У нас также есть возможность использовать нотацию Либница , и мы увидим, как это будет выглядеть со временем. Пока мы будем придерживаться обозначений Лагранжа.

    Вот графики исходной функции и ее первой производной:


    Графики функции ƒ ( x ) = 4 x 4 -2 x 3 -12 x 2 и ее первой производной


    Мы снова применим основные правила дифференцирования, чтобы найти производную функции y = 16 x 3 – 6 x 2 -24 x .Это даст нам вторую производную исходной функции, которую мы можем записать как:

    ƒ ′ ′ ( x ) = 48 x 2 – 12 x – 24

    Вот графики исходной функции и ее первой и второй производных:


    Графики функции ƒ ( x ) = 4 x 4 -2 x 3 -12 x 2 и ее первой и второй производных


    Мы также можем найти производную производной от производной функции.Мы называем эту производную третьей производной. Мы находим третью производную, взяв производную от y = 48 x 2 -12 x -24 (вторая производная), что дает нам:

    ƒ ′ ′ ′ ( x ) = 96 x – 12

    Мы даже можем взять производную от третьей производной (четвертая производная , ), которая равна:

    ƒ (4) ( x ) = 96

    Третья производная дает линейный график.Четвертая производная – постоянное значение, поэтому результирующий график также является линейным, но на этот раз он параллелен оси x . Обратите внимание, что начиная с четвертой производной и далее, мы обозначаем порядковый номер с помощью числа с надстрочным индексом, заключенного в круглые скобки. Сделаем это для наглядности – число просто читать легче, чем длинную строку простых символов. Использование круглых скобок гарантирует, что число не будет ошибочно принято за показатель степени. Хотя нас обычно не интересуют производные, которые образуют линейные графики, мы приводим графики третьей и четвертой производных ниже для полноты картины.Обратите внимание, что пятая производная , являющаяся производной константы, будет равна нулю .


    Третья и четвертая производные функции ƒ ( x ) = 4 x 4 – 2 x 3 – 12 x 2 – линейные функции


    Любые производные за пределами первой производной функции называются производной более высокого порядка или производной более высокого порядка .Вам вполне может быть интересно, и вполне разумно, почему нас будет интересовать производная от производного инструмента или производная от производного инструмента. Мы, конечно, знаем, что первая производная функции сообщает нам, насколько быстро функция изменяется – то есть наклон функции – для любого значения x , для которого функция определена. Другими словами, он сообщает нам мгновенную скорость изменения функции. Точно так же вторая производная сообщает нам, насколько быстро изменяется первая производная функции (которая сама по себе является функцией).Это скорость изменения скорости изменения исходной функции. Третья производная сообщает нам, насколько быстро изменяется вторая производная функции. Это скорость изменения скорости изменения скорости изменения исходной функции.

    Чем выше порядок производной, тем труднее понять, что на самом деле представляет собой производная. К счастью, нам редко приходится иметь дело с чем-то более высоким, чем третья производная.Мы уже знаем, что первая производная дает нам скорость изменения функции, но вторая и третья производные тоже могут быть весьма полезными. Мы можем использовать вторую производную, например, чтобы помочь нам определить потенциальные максимумы и минимумы функции. Но это мы обсудим в другом месте. Производные высшего порядка также находят практическое применение в таких областях, как физика. Классический пример – скорость (или скорость ) по сравнению с ускорением .Представьте себе какой-то объект, движущийся с постоянной скоростью ноль-два метра в секунду (0,2 м / с) по прямой. Если мы построим график смещения-время (т.е. график пройденного расстояния в зависимости от прошедшего времени), он будет линейным, как мы можем видеть ниже.


    График смещения-времени для объекта, движущегося с постоянной скоростью 0,2 м / с


    Вы, вероятно, можете придумать множество примеров ситуаций, в которых объекты движутся с некоторой (более или менее) постоянной скоростью.Автомобиль, движущийся по автомагистрали, может поддерживать постоянную скорость в течение значительного времени, если на нем не слишком много движения. Коммерческие пассажирские самолеты на дальних рейсах обычно курсируют с постоянной скоростью в течение нескольких часов после достижения необходимой высоты. Но как неподвижный объект приобретает постоянную скорость? Он должен пройти ускорение . Если вы не прожили всю свою жизнь в глухой пустыне, вы, вероятно, видели поезд, отходящий от станции, или машину, отъезжающую от светофора.Сначала машина движется медленно. Затем он постепенно набирает скорость (ускоряется), пока не будет достигнута необходимая скорость.

    Предположим, мы снова построим график смещения-времени, на этот раз для поезда, покидающего станцию. Предположим, что поезд испытывает постоянное ускорение со скоростью два метра в секунду в секунду (0,2 м / с 2 ), что дает нам следующую функцию смещения:

    Вот график смещения-времени:


    График смещения-времени для объекта с постоянным ускорением 0.2 м / с 2


    Если мы перепишем функцию, чтобы избавиться от дроби, мы получим:

    ƒ ( x ) = 0,01 x 2

    Теперь, поскольку скорость – это скорость изменения смещения, первая производная функции смещения должна дать нам функцию скорости. Если мы продифференцируем ƒ ( x ) = 0.01 x 2 получаем:

    ƒ ′ ( x ) = 0,02 x

    И поскольку ускорение – это скорость изменения скорости, производная функции скорости должна давать нам функцию ускорения. Если мы продифференцируем ƒ ′ (x) = 0,02 x , мы получим:

    ƒ ′ (ƒ ′ ( x )) = 0,02

    Однако, поскольку мы знаем, что производная от производной функции является второй производной функции, мы можем записать это более элегантно как:

    ƒ ′ ′ ( х ) = 0.02

    Такой результат, конечно, дает нам разгон. Итак, мы можем ясно видеть, что вторая производная функции смещения-времени для тела, движущегося с равномерным ускорением, даст нам фактическую скорость ускорения.

    Вы могли заметить, что все функции, которые мы рассмотрели до сих пор, были полиномиальными функциями . Мы видели, что дифференцирование функции ƒ ( x ) = 4 x 4 -2 x 3 -12 x 2 , которая является полиномиальной функцией четвертой степени, дало нам первую производную функция ƒ ′ ( x ) = 16 x 3 – 6 x 2 -24 x , который является полиномом третьей степени.Вторая производная функция ƒ ′ ′ ( x ) = 48 x 2 -12 x -24 является полиномиальной функцией второй степени, а третья производная ƒ ′ ′ ′ ( x ) = 96 x -12 – полиномиальная функция первой степени (т. е. линейная). Дифференцирование полиномиальной функции степени n всегда приводит к полиномиальной функции степени n – 1, если n не равен нулю, и в этом случае функция является константой, а производная будет равна нулю.Поскольку ноль также является константой, нет смысла пытаться найти производные более высокого порядка.

    Не все функции работают так хорошо. Рассмотрим следующую рациональную функцию :

    ƒ ( x ) = x 2 – 7
    x + 3

    Взяв первую производную, получаем:

    ƒ ′ ( x ) = 2 x x 2 – 7
    x + 3 ( x + 3) 2

    Взяв вторую производную, получаем:

    ƒ ′ ′ ( x ) = 2 4 x + 2 · ( x 2 – 7)
    x + 3 ( x + 3) 2 ( x + 3) 3

    Итак, как вы можете видеть, вместо упрощения высшие производные рациональной функции могут становиться все более сложными.С другой стороны, тригонометрические функции синус и косинус демонстрируют совершенно разное поведение. В каждом случае последовательность производных, как и сама функция, является циклической. Чтобы проиллюстрировать этот момент, мы представляем здесь синусоидальную функцию и ее первые четыре производные:

    ƒ ( x ) = sin ( x )

    ƒ ′ ( x ) = cos ( x )

    ƒ ′ ′ ( x ) = -sin ( x )

    ƒ ′ ′ ′ ( x ) = -cos ( x )

    ƒ (4) ( x ) = sin ( x )

    Начиная с четвертой производной, весь цикл просто повторяется по мере увеличения порядкового номера.Вы, вероятно, видите, что функция косинуса будет вести себя точно так же. Последовательность будет точно такой же. Единственная разница в том, что на этот раз он начинается с cos ( x ). Вот функция косинуса и ее первые четыре производные.

    ƒ ( x ) = cos ( x )

    ƒ ′ ( x ) = -sin ( x )

    ƒ ′ ′ ( x ) = -cos ( x )

    ƒ ′ ′ ′ ( x ) = sin ( x )

    ƒ (4) ( x ) = cos ( x )

    Прежде чем мы оставим эту тему, нам, возможно, следует сказать немного больше об обозначениях.Мы упоминали ранее, что мы можем использовать нотацию Либница для обозначения высших производных, а не нотацию Лагранжа, которую мы использовали до сих пор. Предположим, у нас есть некоторая функция ƒ ( x ). Как мы видели в другом месте на этих страницах, мы можем выразить первую производную от ( x ) следующим образом:

    Мы можем получить выражение для второй производной от ƒ ( x ) следующим образом:

    45 (ƒ x ))
    ( d y ) ′ = d ( d y ) = d 2 y
    d x d x d x d x 2

    После этого третья производная от ( x ) будет записана как:

    Четвертая производная от ( x ) также будет:

    Надеюсь, вы видите образец, который здесь возникает, поэтому мы не будем вдаваться в подробности, расширяя список дальше.Вообще говоря, форма обозначений часто является вопросом личного выбора. Однако могут быть ситуации, в которых та или иная форма более уместна или просто более удобна. Естественно, если ваш наставник просит вас использовать определенную форму записи, вы должны это сделать.


    Эскиз кривой

    В процессе построения кривой выполняются следующие шаги:

    1.Домен

    Найдите область определения функции и определите точки разрыва (если есть).

    2. Перехваты

    Определите точки пересечения \ (x- \) и \ (y – \) функции, если это возможно. Чтобы найти точку пересечения \ (x – \), мы устанавливаем \ (y = 0 \) и решаем уравнение для \ (x. \). Точно так же мы устанавливаем \ (x = 0 \), чтобы найти \ (y- \ ) перехват. Найдите интервалы, в которых функция имеет постоянный знак \ (\ left ({f \ left (x \ right) \ gt 0} \ right. \) И \ (\ left. {F \ left (x \ right) \ lt 0} \ вправо).\)

    3. Симметрия

    Определите, является ли функция четной, нечетной или ни одной, и проверьте периодичность функции. Если \ (f \ left ({- x} \ right) = f \ left (x \ right) \) для всех \ (x \) в области, то \ (f \ left (x \ right) \) является четный и симметричный относительно оси \ (y – \). Если \ (f \ left ({- x} \ right) = -f \ left (x \ right) \) для всех \ (x \) в домене, то \ (f \ left (x \ right) \) нечетное и симметричное относительно начала координат.

    4. Асимптоты

    Найдите вертикальную, горизонтальную и наклонную (наклонную) асимптоты функции.\ prime \ left (x \ right) \) и найдите критические точки функции. (Помните, что критические точки – это точки, в которых первая производная равна нулю или не существует.) Определите интервалы увеличения и уменьшения функции с помощью теста первой производной.

    6. Локальный максимум и минимум

    Используйте первый или второй производный тест, чтобы классифицировать критические точки как локальный максимум или локальный минимум. 2}} \ right] + 2 – \ frac {{2 \ sqrt 3}} {3} = \ cancel {1} – \ sqrt 3 + \ cancel {1} ​​- \ frac {{\ sqrt 3}} {9} – \ cancel {3} + 2 \ sqrt 3 – \ cancel {1} ​​+ \ cancel {2} – \ frac { {2 \ sqrt 3}} {3} = \ frac {{9 \ sqrt 3 – \ sqrt 3 – 6 \ sqrt 3}} {9} = \ frac {{2 \ sqrt 3}} {9} \ приблизительно 0 , 38; \]

    Аналогичным образом находим, что

    \ [y \ left ({1 + \ frac {{\ sqrt 3}} {3}} \ right) = – \ frac {{2 \ sqrt 3}} {9} \ приблизительно -0,38.\]

    Таким образом, функция имеет локальный максимум в точке

    .

    \ [\ left ({1 – \ frac {{\ sqrt 3}} {3}, \ frac {{2 \ sqrt 3}} {9}} \ right) \ приблизительно \ left ({0,42; \ ; 0,38} \ вправо). \]

    Соответственно, локальный минимум достигнут в точке

    .

    \ [\ left ({1 + \ frac {{\ sqrt 3}} {3}, – \ frac {{2 \ sqrt 3}} {9}} \ right) \ приблизительно \ left ({1,58; \; – 0,38} \ вправо) \]

    Интервалы увеличения / уменьшения следуют из рисунка \ (1a. \)

    Рассмотрим вторую производную:

    \ [y ^ {\ prime \ prime} \ left (x \ right) = \ left ({3 {x ^ 2} – 6x + 2} \ right) ^ \ prime = 6x – 6; \]

    \ [y ^ {\ prime \ prime} \ left (x \ right) = 0, \; \; \ Rightarrow 6x – 6 = 0, \; \; \ Стрелка вправо x = 1.2} = \ left ({x + 2} \ right) \ left ({2x – \ cancel {2} + x + \ cancel {2}} \ right) = 3x \ left ({x + 2} \ right) . \]

    Стационарных точек

    \ [у ‘\ влево (х \ вправо) = 0, \; \; \ Rightarrow 3x \ left ({x + 2} \ right) = 0, \; \; \ Rightarrow {x_1} = 0, \; {x_2} = – 2. \]

    Производная меняет свой знак, как показано на рисунке \ (3a. \). Следовательно, \ (x = -2 \) – точка максимума, а \ (x = 0 \) – точка минимума. В этих экстремальных точках функция имеет следующие значения:

    \ [y \ left ({- 2} \ right) = – 4, \; \; y \ left (0 \ right) = 0.3}}} = 0, \; \; \ Rightarrow {x_1} = – \ sqrt 3, \; {x_2} = \ sqrt 3. \]

    При прохождении через эти точки вторая производная меняет знак. Следовательно, обе точки являются точками перегиба. Функция строго выпуклая вниз в интервалах \ (\ left ({- \ infty, – \ sqrt 3} \ right) \) и \ (\ left ({\ sqrt 3, + \ infty} \ right) \) и соответственно, строго выпукло вверх на отрезке \ (\ left ({- \ sqrt 3, \ sqrt 3} \ right).

Оставить комментарий